Preview only show first 10 pages with watermark. For full document please download

General Surgery Reviewer

gs

   EMBED


Share

Transcript

Surgery Reviewer 1. A man sustained severe blunt injuries in a car crash twelve hours ago. The activity of this hormone is now expected to be decreased in this patient: a. insulin b. cortisol c. epinephrine d. aldosterone (FOLD/COVER IF YOU DON’T WANNA SEE THE ANSWERS YET) A (Classification-Application; Source- Schwartz’s th textbook of Surgery 8  edition: pp 25-29) 2. A young woman in debilitated by pruritus and burning sensation from her pre-sternal keloid. The recommended initial treatment is: a. topical application of silicone sheets b. intralesional corticosteroid injection c. surgical excision d. low-dose radiation B (Classification-Recall; Source- Schwartz’s textbook of th Surgery 8  edition: pp. 241-242) 3. A patient is placed on NPO in preparation for an elective major a bdominal surgery. The body’s preferred initial fuel source during the fasting state is: a. hepatic glycogen b. skeletal muscle glycogen c. muscle protein d. body fat A (Classification-Recall; Source- Schwartz’s textbook of th Surgery 8  edition: p. 24) 4. A 48-year-old woman has prolonged ileus after surgery for an obstructed duodenal ulcer. The problem is probably due to: a. hypochloremia b. hypocalcemia c. hypomagnesemia d. hypokalemia 5. A man with chronic prepyloric ulcer appears weak after repeated bouts of non-bilious vomiting over the past three days. Fluid therapy should be started using: a. Normosol M Lactated Ringer’s Solution b. c. Normal Saline Solution d. Hypertonic Saline Solution D (Classification-Application; Source- Schwartz’s th textbook of Surgery 8  edition: pp 48-49) 6. A multiply injured patient has persistently low urine output. The oliguria is most likely due to pre-renal failure rather than acute tubular necrosis if the tests reveal: a. low urine specific gravity b. low urinary excretion of sodium c. low BUN/creatinine ratio d. low creatinine clearance B (Classification-Application; Source- Schwartz’s th textbook of Surgery 8  edition: pp 348-349) 7. A woman admitted to the ICU for severe acute pancreatitis begin to manifest paresthesia of the face and extremities, muscle cramps, and a positive Chvostek sign. These are probably due to: a. hypocalcemia b. hypokalemia c. hypomagnesemia d. hypophosphatemia A (Classification-Application; Source- Schwartz’s th textbook of Surgery 8  edition: p. 50) 8. Increased nutritional support appears to aggravate tachypnea in a septic patient with respiratory failure. Lessening the amount of this substrate may alleviate the problem: a. carbohydrate b. fat emulsion c. essential amino acids d. non-essential amino acids A (Classification-Recall; Source- Schwartz’s textbook of th Surgery 8  edition: pp 28-29) 9. Administration of this amino acid is expected to be beneficial to a patient suffering from damage to intestinal mucosa due to adjuvant chemotherapy for esophageal cancer. a. Arginine b. Valine c. Leucine d. Glutamine D (Classification-Recall; Source- Schwartz’s textbook of th Surgery 8  edition: p. 38) 10. Early supplementation of this vitamin is recommended to promote wound repair in a patient with extensive second degree flame burns a. A b. C c. D d. E B (Classification-Application; Source- Schwartz’s th textbook of Surgery 8  edition: p. 210) 11. A diabetic 41-year-old man is admitted for necrotizing fasciitis in the perineum. A high dose of this antibiotic generally included in the initial antimicrobial therapy because of concern for clostridial pathogens. C (Classification-Recall; Source- Schwartz’s textbook of C (Classification-Application; Source- Schwartz’s th textbook of Surgery 8  edition: pp 51-52) a. b. c. d. Metronidazole Vancomycin Penicillin G Aminoglycoside th Surgery 8  edition: p.122) 12. Transfusion of properly cross-matched blood is begun on a man admitted for massive bleeding from erosive gastritis and thirty minutes later, he develops urticaria and fever. This should be administered to the patient. a. anti-histamine b. Mannitol c. Furosemide d. Sodium Bicarbonate A (Classification-Application; Source- Schwartz’s th textbook of Surgery 8  edition: pp 79-80) 13. A patient under anticoagulation therapy using warfarin, who is set to undergo surgery for acute cholecystitis, has decreased prothrombin concentration. Warfarin can be reversed by parental dose of a. Protamine sulfate b. Vitamin K c. EACA d. Hydroxyurea 14. Arterial blood gas analysis is performed on a patient just admitted with diagnosis of severe acute pancreatitis. This reveals a pH of 7.30 and levels of bicarbonate of pCO2. The most urgent part of management is a. Volume resuscitation b. Intravenous bicarbonate c. Calcium infusion d. Mechanical ventilation 15. Seven days after surgery for perforated appendicitis, the primarily closed incision is noted to be erythematous, slightly swollen and tender. The appropriate treatment is: a. Local heat therapy b. Topical antibiotics c. New systems antibiotics d. Incision and drainage 16. A man is brought to the E.R. with blood spurting from a hacking wound is the distal right thigh. He is alert and has a systolic BP of 100 mmHg. What is the initial management step? a. Apply direct pressure on the wound with sterile gauze b. Apply digitial pressure on proximal femoral artery c. Apply a thigh tourniquet above the wound d. Open the wound and clamp the bleeders B (Classification  – Application; Source Schwartz’s th Textbook of Surgery (8  edition): pp 73-74) 17. A young man presents to the E.R with a stab wound in the left chest. Examination reveals subcutaneous emphysema and absent breath sounds on the left chest; the trachea is shifted to the right. What is the probable diagnosis a. Massive hemothorax b. Tension pneumothorax c. Cardiac tamponade d. Flail chest 18. A 58-year old woman NSAID therapy for arthritis has a 6 hr history of epigastric pain that has progressively become severe. Her abdomen is distended, rigid and diffusely tender. This diagnostic test should be done first: a. Upright chest X-ray b. Esophagogastroduodenoscopy c. Abdominal ultrasound d. Abdominal CT scan B (Classification  – Application; Source Schwartz’s th Textbook of Surgery (8  edition): pp 131) 19. After aggressive fluid therapy, a trauma victim shows a systolic BP of 110 mmHg, cold extremities, rapid but strong peripheral pulse, and a central venous pressure of 12 cmH20. The patient has: a. Increased systemic vascular resistance b. Decreased stroke volume index c. Decreased cardiac index d. Excessive cardiac preload A (Classification  – Application; Source Schwartz’s th Textbook of Surgery (8  edition): pp 132) 20. An elderly man admitted with a diagnosis of sigmod volvulus has a markedly distended, non-tender abdomen and hyperactive bowel sounds. The initial treatment is: a. Endoscopic detorsion b. Detorsion via laparotomy c. Decompressing transverse loop colostomy d. Resection of involved segment of colon 21. Pancreatic necrosis is suspected in a 49-year-ol woman who has not improved despite 3 days of intensive care for acute pancreatitis. The diagnosis is best established though: a. C-reactive measurement b. Abdominal ultrasound c. Contrast-enhanced CT scan d. CT-guided percutaneous biopsy A (Classification  – Application; Source Schwartz’s th Textbook of Surgery (8  edition): pp 1098) A (Classification  – Application; Source Schwartz’s th Textbook of Surgery (8  edition): pp 50-51) D (Classification  – Application; Source Schwartz’s th Textbook of Surgery (8  edition): pp 119-120) A (Classification  – Application; Source Schwartz’s th Textbook of Surgery (8  edition): pp 74) A (Classification  – Application; Source Schwartz’s th Textbook of Surgery (8  edition): pp 959-960) C (Classification  – Application; Source Schwartz’s th Textbook of Surgery (8  edition): pp 1234-1238) 22. A chronically constipated 67-year-old woman presents with acute left lower quadrant (LLQ) abdominal pain, LLQ direct and rebound tenderness, and fever. The appropriate diagnostic examination is: a. Transvaginal ultrasound b. Abdominal CT scan c. Proctosigmoidoscopy d. Barium enema B (Classification  – Application; Source Schwartz’s th Textbook of Surgery (8  edition): pp 1082-1083) 23. After an elective hemicolectomy for colon cancer, a 78-year-old man remains hypotensive and tachycardic. He has distended neck veins, cold skin, oliguria, and elevated central venous pressure. He apparently is suffering from this type of shock: a. Hypovolemic b. Cardiogenic c. Septic d. Neurogenic B (Classification  – Application; Source Schwartz’s th Textbook of Surgery (8  edition): pp 95-100) 24. For the past 6 weeks, a 67-year-old has been asymptomatic except for constipation after a course of antibiotic therapy for left lower quadrant abdominal pain. He should undergo: a. CEA determination b. Barium enema c. Colonoscopy d. Abdominal CT scan C (Classification  – Application; Source Schwartz’s th Textbook of Surgery (8  edition): pp 1082-1083) 25. A 32-year- olf man with a week’s history of fever presently has right lower quadrant abdominal pain and tenderness and bloody diarrhea. The most probable diagnosis is: a. typhoid ileitis b. amebic colitis c. ileocecal TB d. intussusception A (Classification  – Application; Source  – Schwartz’s th Textbook of Surgery (8  edition): pp. 1049-1050) 26. A 75 year old woman in the ICU after undergoing cholecystectomy for acute cholecystitis is hypotensive and tachycardic. Pulmonary capillary wedge pressure (PSCP) is elevated to 18mmHg, and cardiac output is 3L/min. Hershock is best described as which of the following? a. hypovolemic shock b. septic shock c. cardiogenic shock d. anaphylactic shock C (Classification  – Recall; Source  – Schwartz’s Textbook th of Surgery (8  edition): pp.) 27. A 19 year old man is brought to the emergency department with a stab wound at the base of the neck (zone 1). The most important concern for the patients with such injuries is which of the following? a. upper extremity ischemia b. cerebral infarction c. exsanguinating hemorrhage d. mediastinitis C (Classification  – Recall; Source  – Schwartz’s Textbook th of Surgery (8  edition): pp.) 28. In septic shock, which of the following is true? a. The mortality rate is between 10 and 20% b. Gram negative organisms are involved exclusively c. The majority of patients are elderly d. The most common source of infection is the alimentary tract. C (Classification  – Recall; Source  – Schwartz’s Textbook th of Surgery (8  edition): pp.) 29. The injury most often missed by selective nonoperative management of abdominal stab wounds is to which of the following? a. colon b. spleen c. ureter d. diaphragm D (Classification  – Recall; Source  – Schwartz’s Textbook th of Surgery (8  edition): pp.) 30. A 63-year-old male tobacco smoker has a 1.5 cm non-healing ulcer in his lower lip. A nodular lesion is palpable deep to the ulcer. The most likely diagnosis is: a. squamous cell carcinoma b. keratoacanthoma c. malignant fibrous histiocytoma d. verrucous carcinoma A (Classification  – Application; Source  – Schwartz’s th Textbook of Surgery (8  edition): pp. 518) 31. A 43-year-old-man presents with nasal obstruction and occasional occasional epistaxis. Imagi ng studies combined with endoscopic bipsy have led to the diagnosis of nasopharyngeal cancer. The standard treatment is: a. intracavitary radiation b. external beam radiation c. combined chemotherapy and radiation d. surgical resection C (Classification  – Application; Source  – Schwartz’s th Textbook of Surgery (8  edition): pp. 1040-1050) 32. Biopsy of a slow growing infraauricular mass reveals a benign pleomorphic adenoma that is confined to the superficial lobe of the parotid gland. The standard treatment is: a. tumor enucleation b. tumor excision with 2mm margin c. superficial parotidectomy d. surgical resection 33. A 58 year old woman undergoes excision biopsy of a tumor in the left posterior triangle of her neck. Histology suggests that this is a metastatic cancer. What is the most likely site of the primary tumor? a. Ovary b. Adrenal Gland c. Kidney d. piriform fossa C (Classification  – Recall; Source  – Schwartz’s Textbook th of Surgery (8  edition): pp 538-540) (Classification  – Recall; Source  – Schwartz’s Textbook th of Surgery (8  edition): pp) 34. A 59 year old woman has discomfort in the posterior part of her tongue. A biopsy confirms that the lesion is a carcinoma. What is true in carcinoma of the posterior third of the tongue? a. lymphoid tissue is absent b. lymphoid gland spread is often encountered c. there is an excellent prognosis d. the tissue is well differentiated B (Classification  – Recall; Source  – Schwartz’s Textbook th of Surgery (8  edition): pp) 35. A 60 year old man has a 3 cm nodular nodular lesion with central ulceration in his left cheek. Section biopsy proves this to be basal cell carcinoma. This is best managed with: a. external beam radiation b. topical 5-fluouracil c. electrodessication d. excision with 2-4mm margin D (Classification  – Application; Source  – Schwartz’s th Textbook of Surgery (8  edition): pp 439-440) 36. This is the most common aggressive primary malignant bone tumor in adolescent and occurs in methaphyseal area of long bones with high incidence of pulmonary metastases: a. Chrondrosarcoma b. Osteosarcoma c. Fibrosarcoma d. Adamantinoma B (Classification  – Recall; Source  – Schwartz’s Textbook th of Surgery (8  edition): pp) 37. A 65-year-old farmer presents with a 1.5-cm ulcerated lesion on the middle third of his lower lip. The lesion has been present for 4 months and is not painful. No lymph nodes are palpable in the patient’s neck. The most likely diagnosis is; a. Squamous cell carcinoma b. Basal cell carcinoma c. Herpes simplex d. Keratoacanthoma A (Classification  – Application; Source  – Principles of th Surgery by Schwartz, Spencer, Shires 7  edition Volp pp 521-522 38.A 65-year-old patient who spends summer in Baguio City presents with a painless, ulcerated lesion on the right cheek. The lesion has been present for one year. Physical examination of the patient’s neck reveals no lymph node enlargement. The most likely diagnosis is: a. pyogenic granuloma b. melanoma c. basal cell carcinoma d. squamous cell carcinoma C (Classification  – Application; Source  – Principles of th Surgery by Schwartz, Spencer, Shires 7  edition Vol 1, p 522) 39. A 60-year-old woman has a 10 day history of cough and fever. Imaging studies show multiple fluid loculations in the right chest cavity with an estimated volume of 500 ml. Thoracentesis draws purulent material. The best treatment strategy now is: a. pigtail catheter drainage of the empyema b. video assisted thorascopic drainage with deloculation c. insertion of multiple thoracostomy tubes d. early thoracotomy and drainage B (Classification  – Application; Source  – Principles of th Surgery by Schwartz, Spencer, Shires 7  edition Vol 1, pp 601-602) 40. A 50-year-old woman presents with a 3 cm solitary pulmonary nodule in the left upper lobe. She has a history of total thyroidectomy for papillary cancer at the age of 38 years. The main consideration is: a. TB granuloma b. hamartoma c. primary lung cancer d. metastatic thyroid cancer C (Classification  – Application; Source  – Schwartz’s th Textbook of Surgery (8  edition): pp 556-557) 41.A 45-year-old woman presents with weakness towards the afternoon, shortness of breath, and ptosis. CT scan reveals an anterior mediastinal mass. The most likely diagnosis is: a. intrathoracic goiter b. non-Hodgkin’s lymphoma c. non-seminomatous germ cell tumor d. thymoma D (Classification  – Application; Source  – Schwartz’s th Textbook of Surgery (8  edition): pp 593-594) 42. A man with malignant pleural effusion in association with an inoperable lung cancer may benefit from the instillation of sclerosing agent into the pleural cavity, once the lungs are almost fully expanded. An accepted sclerosant is: a. talc b. gentamicin c. penicillin G d. Vitamin A (Classification  – Application; Source  – Schwartz’s th Textbook of Surgery (8  edition): pp 600-601) 43. A 20-year-old woman consults for a 2-cm mass in her left breast. The mass is movable, non-tender, and has a rubbery consistency and smooth borders. The probable diagnosis is: a. fibroadenoma b. fibrocystic disease c. carcinoma d. cytosarcoma phyllodes A (Classification  – Application; Source  – Schwartz’s th Textbook of Surgery (8  edition): pp 463-464) 44. A 16-year-old girl is bothered by cyclical premenstrual pain her breasts. She has 0.5-1.0 cm nodularities in the upper outer quadrant of both breasts. What management advice should be given to her? a. Observation b. breast ultrasound c. mammography d. aspiration biopsy A (Classification  – Application; Source  – Schwartz’s th Textbook of Surgery (8  edition): pp 465-466) 45. An asymptomatic 40-year-old woman with jno palpable breast mass undergoes mammography. This reveals clustered microcalcifications in the lower outer quadrant of her left breast. The probable diagnosis is: a. florid hyperplasia b. sclerosing adenosis c. atypical ductal hyperplasia d. ductal carcinoma in situ D (Classification  – Application; Source  – Schwartz’s th Textbook of Surgery (8  edition): pp 472-473)D 46. A 45-year- old woman with “lumpy” breasts undergoes mammography. Mammographic findings are suggestive of high grade ductal carcinoma in situ confined to the upper outer quadrant of her right breast. How should she be managed? a. lumpectomy alone b. lumpectomy and radiotheraphy c. mastectomy d. mastectomy and axillary node sampling B (Classification  – Application; Source  – Schwartz’s th Textbook of Surgery (8  edition): p 481) 47. A 45-year-old woman has a recent onset bloody nipple discharge from her right breast. No palpable breast mass is noted. What diagnostic test is indicated? a. breast ultrasound b. mammography c. ductography d. cytology of discharge B (Classification  – Application; Source  – Schwartz’s th Textbook of Surgery (8  edition): p 493) 48. A premenopausal woman undergoes modified radical mastectomy for a 3-cm breast cancer. No axillary node and distant metastases are detected. Test for this biomarker is currently recommended to facilitate the selection of adjuvant chemotheraphy. a. c-fos b. c-myc c. p53 d. AHER2/neu D 49. A 63-year old man with chronic atrial fibrillation has sudden onset of pain, weakness and paresthesia in his left leg, which appears cool, cyanotic and without femoral and distal pulses. The right leg has normal pulses. The most likely diagnosis is: a. arterial embolism b. aortoillac thrombosis c. Buerger’s disease d. Raynaud’s disease A 50. Coronary angiogram reveals a triple-vessel disease in a diabetic 55-year-old man presenting with unstable angina. The recommended treatment is: a. catherter-directed thrombolysis b. balloon angioplasty c. coronary artery bypass grafting d. transmyocardial revascularization C 51. A 41-year old male chronic smoker has a 2-year history of bilateral foot claudication. He now has ulceration rd nd th of the tip of the right 3  toe and left 2  and 4  toes. Popliteal pulses are palpable but the posterior tibial and dorsalispedis pulses are bilaterally absent. The most important step in management is: a. cessation of smoking b. infusion of prostaglandin c. administration of indomethacin A d. immediate surgical correction of PDA 53. A 3-year-old girl presents with cyanotic spells that have increased in severity and frequency since birth. Xray shows a boot-shaped heart, ECG reveals right ventricular hypertrophy. The most probable diagnosis is: a. Truncusarteriosis b. tetralogy of Fallot c. transposition of the great arteries Ebstein’s anomaly d. B 54. A 54-year-old man presents with hematemesis after a bout of vomiting and retching. Endoscopy shows linear mucosal tears at the gastroesophageal junction.The diagnosis is: a. Mallory-Weiss syndrome b. Boerhaave’s syndrome Menether’s syndrome c. d. Dieulafoy’s syndrome A 55. After truncalvagotomy and antrectomy with gastrojejunostomy for an obstructed duodenal ulcer, the patient complains of diaphoresis, weakness and abdominal discomfort followed by diarrhea several minutes after meals. The symptoms are suggestive of: a. post vagotomy diarrhea b. bile reflux gastritis c. afferent loop syndrome d. dumping syndrome D 56. A woman undergoes total gastrectomy for a huge proximal gastric carcinoma. To protect her from developing anemia, she must be given a regular parenteral dose of: a. folic acid b. ferrous sulfate c. vitamin B12 d. transferrin C 57. A few weeks after recovering from severe pancreatitis, a 34-year-old man has developed a pancreatic pseudocyst with mature wall that is pressing on his stomach. Aspiration fails to relieve the pressure symptoms. The indicated treatment is: a. administration of somatostatin b. internal drainage (cystogastrostomy) c. external drainage d. excision of pseudocyst B 58. ERCP reveals a penampullary cancer in a 64-year-old man admitted for jaundice. No metastasis is detected. No co-morbid conditionas are identified. How should he be managed? a. radical excision of the head of pancreas and duodenum b. local excision and adjuvant chemotheraphy c. external beam radiation d. stenting and chemotheraphy A 59. A 60-year-old man with no previous operation has a 5-day history of inability to pass flatus or feces, cramping abdominal pain, and progressive abdominal distention. The principal diagnostic consideration is: a. midgut volvulus b. sigmoid volvulus c. colorectal cancer d. ileocecal tuberculosis A (Classification-Application; Source- Schwartz’s th textbook of Surgery 8  edition: pp 25-29) 60. A man who has an obstructed rectosigmoid cancer in association with a competent ileocecal valve is liable to develop perforation of: a. sigmoid colon b. splenic flexure c. hepatic flexure d. cecum D (Classification-Application; Source- Schwartz’s th textbook of Surgery 8  edition: pp 1089-1090) 61. The presence of tenesmus, decrease in calibre of stools and occasional bloody-mucoid diarrhea in a 67year-old man is suggestive of: a. rectal carcinoma b. amoebic colitis c. intussusceptions d. ileocecal tuberculosis A (Classification-Application; Source- Schwartz’s th textbook of Surgery 8  edition: pp) 62. Diagnostic tests show an ileocolic intussusceptions in a 38-year-old man, whoo has no signs of strangulation. What is recommended for this patient? a. expectant treatment b. endoscopic decompression c. hydrostatic reduction d. exploratory laparotomy (Classification-Application; Source- Schwartz’s th textbook of Surgery 8  edition: pp) 63. A 63-year old man is presently asymptomatic after medical treatment for LLQ pain and tenderness, He should undergo: a. CEA determination b. Colonoscopy c. Barium enema d. Abdominal CT scan 64. A 56-year-old woman taking NSAIDs for chronic arthritis experienced severe epigastric pain 6 hours ago. Her abdomen is now diffusely tender with board-like rigidity. The appropriate initial diagnostic test is: a. chest upright xray b. plain abdominal supine xray c. abdominal ultrasound d. abdominal CT scan B (Classification-Application; Source- Schwartz’s th textbook of Surgery 8  edition: pp) (Classification-Application; Source- Schwartz’s th textbook of Surgery 8  edition: pp) 65. Despite 3 months of medical treatment for posterior midline anal fissure, the man’s fissure-related anal painduring and after defecation has become excruciating. The appropriate treatment is: a. excision of fissure b. botulinum toxin injection c. anal divulsion d. internal sphincterotomy D (Classification-Application; Source- Schwartz’s th textbook of Surgery 8  edition: pp 1103-1104) 66. During surgery for a large indirect inguinal hernia, the cecum is noted to form part of the wall of the hernia sac. The patient has this type of hernia: a. sliding Richter’s b. c. interstitial d. Spigelian A (Classification-Application; Source- Schwartz’s th textbook of Surgery 8  edition: 1358) 67. A previously asymptomatic 45-year-old obese woman consults for a non-tender swelling below her right inguinal ligament just lateral to the pubic tubercle. A warranted diagnostic procedure for this patient is: a. fine needle aspiration b. ultrasound c. MRI d. herniography B (Classification-Application; Source- Schwartz’s th textbook of Surgery 8  edition: pp 1364-1366) 68. A 54 year old male taking anti-coagulant for the past 4 weeks developed 3 day history of abdominal pain. On physical examination, there was a palpable mass at the supraumbilical area with direct tenderness. Fothergill sign was positive. What is your impression? a. desmoid tumor b. rectus sheath hematoma c. omental torsion d. omental infarction B (Classification-Application; Source- Schwartz’s th textbook of Surgery 8  edition: pp ) 1. A 65 year-old male who underwent a colon resection of carcinoma 2 years prior to consult is found to have 3 solid nodules approximately 2 cms each in the right and left lobes of the liver. There is no extrahepatic disease detected. The preferred treatment is: a. systemic chemotherapy b. radiofrequency ablation c. microwave coagulation therapy d. surgical resection of metastatic nodule D (Classification-Application; Source- Schwartz’s th textbook of Surgery 8  edition: pp) 2. A 35 year old female with a prolonged intake of contraceptive pills develop right upper quadrant pains. UItrasound shows an isodense 5cm mass in the right lobe of the liver. Sulfur-colloid scan done showed a cold lesion. The patient should undergo/; a. conservative treatment b. resection c. enucleation d. liver transplantation B (Classification-Application; Source- Schwartz’s th textbook of Surgery 8  edition: pp 1089-1090) 69. A 12 year old female complain of 1 day history of epigastric pain which shifted to the right lower quadrant associated with anorexia and vomiting. Physical examination revealed direct and rebound tenderness on the right lower quadrant. With the preoperative diagnosis of acute appendicitis, she was operated on. However, intraoperatively there was blood tinged peritoneal fluid and appendix was noted to be normal. The most probable cause of the patient condition is: a. mesenteric mesenteric cyst b. omental torsion c. ruptured rectus sheath Hematoma d. mesenteric adenitis B (Classification  – Application; Source  – Schwartz’s th pp.) Textbook of Surgery (8  edition): pp.) 70. A 45-year-old is discovered to have solitary 2-cm form nodule in his right thyroid lobe. No associated symptoms are noted; family history is negative for thyroid cancer. Diagnostic work-up should start with: a. thyroid function tests D (Classification  – Application; Source  – Schwartz’s th Textbook of Surgery (8  edition): pp 1413-1416) b. c. d. thyroid ultrasound thyroid scan fine needle aspiration biopsy 71. A 59-year-old man presents with 4-cm firm mass in the left lobe of the thyroid associated with a form occipital mass that is 6cm in its widest diameter, which he has neglected for the past 9 months. Family history is negative for thyroid malignancy. He probably has this type of thyroid cancer: a. Papillary b. Follicular c. medullary d. anaplastic B (Classification  – Application; Source  – Schwartz’s th Textbook of Surgery (8  edition): pp1420-1421) 72. A 49-year-old woman complains of central obesity with moonlike fascies and hirsutism. Her ACTH is markedly elevated which is suppressed by large dose of dexamethasone. The probable pathology is: a. adrenal adenoma b. adrenal carcinoma c. pheochromocytoma d. pituitary adenoma D (Classification  – Application; Source  – Schwartz’s th Textbook of Surgery (8  edition): pp 1455-1458) 73. A 42-year-old man presents with headache, palpitations, and diaphoresis. His urinary metanephrines are elevated. Imaging studies reveal a 3-cm mass in his right adrenal gland. The treatment of choice is: a. Adrenalectomy b. Radiotherapy c. tumor embolization d. chemoradiation A (Classification  – Application; Source  – Schwartz’s th Textbook of Surgery (8  edition): pp 1460-1461) 74. A 67-year-old man is bleeding excessively following a transurethral prostatec tomy. The bleeding is assesses to be due to local plasminogen activation leading to increased fibrinolysis on the raw wound surface. Administration of this drug may be beneficial: a. vitamin K b. heparin c. warfarin d. EACA D (Classification  – Application; Source  – Schwartz’s th Textbook of Surgery (8  edition): p 82) 75. A 60-year-old man presents with slight urinary urgency, frequency, and a decrease in the force of micturition. Digital rectal examination detects no mass in the prostrate. The indicated diagnostic test is: a. prostatic acid phosphatase b. prostate specific antigen c. pelvic ultrasound d. needle biopsy of prostrate A (Classification  – Application; Source  – Schwartz’s th Textbook of Surgery (8  edition): pp 1535-1537) 76. A 35-year-old man presents with right flank pain and microscopic hematuria. Intravenous pyelography reveals a 4 mm radiolucent stone in the distal right ureter. The recommended treatment is: a. alkalinziation of the urine b. extracorporeal shock wave lithotripsy c. stone extraction via ureteroscope d. open ureterolithomy A (Classification  – Application; Source  – Schwartz’s th Textbook of Surgery (8  edition): pp 1546-1549) 77. A 55-year-old man is admitted for gross hematuria associated with left flank plain and anemia. CT scan detects a solid, enhancing mass in the left kidney. The likely diagnosis is: a. renal cell carcinoma b. renal tuberculosis c. renal sarcoma d. renal oncocytoma A (Classification  – Application; Source  – Schwartz’s th Textbook of Surgery (8  edition): pp 1531-1532) 78. A 23-year-old woman sustained blunt head trauma to her right frontoparietal area in a car crash, which caused him to be unconscious for several minutes. Upon arrival at the ER, she is alert and has no neurologic deficit. X-ray shows a linear on-depressed right frontoparietal fracture. An hour after admission, vomiting followed by progressive neurologic deterioration are noted. a. diffuse axonal injury b. cerebral contusion c. acute epidural hematoma d. acute subdural hematoma C (Classification  – Application; Source  – Schwartz’s th Textbook of Surgery (8  edition): p 1619) 79. A 54-year-old woman with no previous neurologic disorder has a new onset seizure without associated gross neurologic deficit. She was treated for breast cancer 10 years ago. If brain metastasis is the main consideration, this is best confirmed by means of: a. A. Electroencephalography b. B. CT scan of the brain with intravenous contrast c. C. MRI of the brain with intravenous contrast d. D. Doppler ultrasound C (Classification  – Application; Source  – Schwartz’s th Textbook of Surgery (8  edition): pp 1633) 80. A 55-year-old man with a history of lung cancer presents with frequent severe headache, occasional vomiting, and weakness of the right upper extremity. What is the recommended treatment in addition to corticosteroids? a. A. intrathecal chemotherapy b. B. whole-brain radiotherapy c. C. stereotactic radiosurgery (gamma knife) d. D. resection of lesion causing arm weakness C (Classification  – Application; Source  – Schwartz’s th Textbook of Surgery (8  edition): p 1633) 81. A young man just admitted for blunt injuries from a car accident is stuporous. CT scan shows cerebral contusion. What should be avoided as it can cause secondary brain injury to this patient through its deleterious effect on intracranial pressure? A. elevation of head B. respiratory acidosis C. hypovolemia even of mild degree D. sedation agitated patient B (Classification – Recall; Source  – Schwartz’s Textbook of Surgery (8th edition): pp. 1613-1614) 82. As a basketball player falls hard on his extended right arm, he feels immediate severe pain with subsequent inability to move his right arm. Swelling and tenderness about the shoulder with loss of deltoid contour are noted. The likely diagnosis is: A. deltoid muscle rupture B. shoulder dislocation C. fracture of proximal humerus D. brachial plexus injury B (Classification  – Application; Source - Schwartz’s Textbook of Surgery (8th edition): pp. 1706-1707) 83. A 10-week-old baby with cleft lip and palate has undergone a successful cleft lip repair. It is recommended that cleft palate repair be done before the baby reaches this age: A. 6 months B. 12 months C. 2 years D. 4 years B (Classification – Recall; Source  – Schwartz’s Textbook of Surgery (8th edition): pp. 1797-1799) 84. A bedridden 61-year-old woman has a significant neurologic recovery from a previous stroke. She has an infected deep pressure ulcer in her sacrum. Once the infection has been controlled by debridement and supportive treatment, the pressure ulcer can be managed with: A. skin graft B. rotation skin flap C. gluteus maximus flap D. microsurgery free flap C (Classification  – Recall; Source  – Schwartz’s Textbook of Surgery (8th edition): pp. 1825-1826) 85. A small thick antecubital scar from a 2nd degree flame burn is constricting movement. To relieve the constriction, the appropriate procedure to refashion the scar is: A. Z-plasty B. advancement flap C. scar excision and skin graft D. deltopectoral flap A (Classification  – Recall; Source  – Schwartz’s Textbook of Surgery (8th edition): pp. 1792-1793) 86. Resection of a sarcoma results in a significant lateral chest wall defect. To minimize pulmonary dysfunction, chest wall reconstruction may utilize: A. rectus abdominis muscle flap B. omental flap C. tensor fascia lata graft D. Marlex mesh D (Classification  – Recall; Source  – Schwartz’s Textbook of Surgery (8th edition): pp. 1823-1824) 87. A 4-week old baby boy presents with frequent non-bilious vomiting. During feeding peristaltic waves are seen in the upper abdomen in association with a palpable mass in the right upper quadrant. The diagnosis is: A. hypertrophic pyloric stenosis B. duodenal atresia C. jejuna atresia D. volvulus neonatorum A  (Classification  – Application; Source  – Schwartz’s Textbook of Surgery (8th edition): pp. 1486-1487) 88. A newborn infant has excessive drooling followed by coughing immediately after feeding. The main diagnostic consideration is esophageal atresia. This can be readily confirmed by: A. inability to pass orogastric tube into the stomach B. water-soluble contrast study (esophagram) C. transesophageal ultrasound D. plain abdominal radiograph A (Classification  – Application; Source  – Schwartz’s Textbook of Surgery (8th edition): pp. 1481-1482) 89. A diagnosis of intussusceptions is made in an infant with paroxysms of crampy abdominal pain and vomiting. The infant has no peritoneal signs and remains hemodynamically stable. The initial treatment of choice is: A. nasogastric decompression B. endoscopic reduction C. pneumatic reduction C (Classification  – Recall; Source  – Schwartz’s Textbook of Surgery (8th edition): pp. 1493-1494) D. hydrostatic reduction with barium 90. A 3-day old infant who has not passed meconium presents with abdominal distention and bilious vomiting. The most likely diagnosis is: A. duodenal atresia B. malrotation C. midgut volvulus D. Hirschsprung’s disease D (Classification  – Application; Source  – Schwartz’s th Textbook of Surgery (8  edition): pp. 1496-1497) 91. A 3-week-old infant is noted to have jaundice in association with acholic stools shortly after birth. A radionuclide scan demonstrates presence of radioisotope in the intestine. This finding rules out the diagnosis of : A. neonatal hepatitis B. biliary atresia C. inspissated bile syndrome D. choledochal cyst B (Classification  – Application; Source  – Schwartz’s Textbook of Surgery (8th edition): pp. 1499-1500) 92. A 67 y/o man has an intraabdominal abscess caused by perforated sigmoid diverticulitis. The most common isolated microorganism is _? A. Escherichia coli B. Bacteroides C. K. pneumonia D.Staphylococcus epidermides B Schwartz’s: surgical infection p.121 93. One of the most common complications of central venous access is ____. A. hemorrhage B. pneumothorax C. wound infection D. air embolus Complication in Surgery p. 337 B Schwartz’s Complication 94. Early postoperative small bowel obstruction is a rare finding and occurs less than 1% of the time. When it does, the most common cause is ____. A. internal hernias B. post-operative adhesions C. infections or abscess D. technical errors B Schwartz’s: Surgical Complications p. 347 95. Which of the following statements is true concerning corrosive injury to the esophagus? A. Acid ingestion is not injurious to the stomach because of its non-acidic pH. B. Ingested caustic agents rapidly pass through the esophagus and stomach into the small intestines. C. Alkaline injury is more destructive than acid injury. D. Acid injury is more destructive than alkaline injury. C Schwartz’s: Esophagus p.909 96. A 27 year old female was brought to the ER with history of suicidal ingestion of household muriatic acid. This patient is best managed by ---A. Give half strength vinegar, lemon juice or orange juice B. Start with milk, egg white or antacids C. Give emetics to induce vomiting D. Sodium bicarbonate is one of the treatment option B Schwartz’s: Esophagus p.910 97. Which of the following statements is true regarding the arterial blood supply of the stomach? A. The right gastric artery, a branch of the superior mesenteric artery supplies the gastric antrum B. Because of the rich intramural collateral vessels, gastric viability may be preserved after ligation of at least two of the four named gastric arteries. C. The largest artery supplying the stomach is the right gastric artery. D. The left gastroepiploic artery is a branch of the celiac trunk. B Schwartz’s: Stomach pp.935-937 98. At a cellular leve, the major stimulants of acid secretion by the gastric parietal cell are the following, except: A. histamine B. prostaglandinE2 C. acetylcholine acetylcholine D. gastrin B Schwartz’s: Stomach p. 943 99. A patient with gastric adenocarcinoma underwent subtotal gastrectomy. Histopath revealed tumor penetrating the serosa, regional lymph nodes are not involved and distant metastatic lesions are not detected. What is the correct tumor stage on TNM staging? A. Stage I B. Stage II C. Stage III-A D. Stage III-B B Schwartz’s: Stomach p. 970 100. A 5-month old baby boy was seen at the ER with complaints of non-bilous vomiting that became increasingly projectile over several days to weeks and cannot tolerate liquid intake. Abdominal examination C Schwartz’s: Pediatric Surgery p. 1486 revealed palpation of typical “olives” in the right upper quadrant and cannot tolerate liquid intake. Abdominal examination revealed revealed palpation of typical “olives” in the right upper quadrant with visible gastric waves. What is the correct diagnosis? A. meconium ileus B. Hirschprung’s disease C. Hypertrophic pyloric stenosis D. Intussusception 101. Which of the following is the most common malignant liver tumor in childhood? A. Hemangioma B. Hepatoblastoma C. Hepatocellular carcinoma D. Mesenchymal hamartoma B Schwartz’s: Pediatric Surgery p. 1512 102. What is the appropriate treatment for an incidental 3cm liver hemangioma found on abdominal CT scan of 37 year old woman? A. No treatment B. Cryoablation C. Selective embolization of left hepatic artery D. Left hepatic lobectomy A Schwartz’s: pp. 1162-1163 103. A 35 y/o male presents with 1 year history of early satiety and right upper abdominal discomfort. CT scan shows a 6x8 cm cystic lesion in the right lobe of the liver. Ultrasound of cyst shows no internal echoes. The appropriate management is: A. cryoablation of cyst B. right hepatic lobectomy C. unroofing of the cyst D. percutaneous aspiration of cyst C Schwartz’s: p. 1159 104. An imaginary line that divides the liver into right and left lobe that runs from the inferior vena cava to the tip of the gallbladder fossa: A. Catlie line B. Calot’s line C. Cantlie’s line D. Charcot’s line C Schwartz’s: p. 1140 105. The diagnostic tool important in evaluating liver injury in a stable blunt trauma patient is: a. Ultrasound Ultrasound c. CT  – Scan b. Abdominal X-ray d. Peritoneal Lavage C Schwartz’s: p. 142 106. The local factor affecting wound healing a. Age b. Smoking C Schwartz’s: p. 235 c. Low oxygen tension d. Trauma 107. The characteristics of keloid scars: a. Arise above the skin level and stay in the confine of the original wound b. Develop within 4 weeks after trauma c. Extend beyond the border of the original wound and rarely regresses spontaneously d. Occur in areas of flexion and extension C Schwartz’s: p. 240 108. A 55 y/o male diagnosed to have chronic pancreatitis, devel oped epigastric mass measuring about 6 cm accompanied by pain and fullness. The diagnosis is: a. Gastric Carcinoma c. Pseudocyst of the pancreas b. Pancreatic Carcinoma d. Acute pancreatitis C Schwartz’s: p. 1256 109. Type of benign solitary pancreatic neoplasm consisting of symptomatic fasting hypoglycemia and profound syncopal episodes: a. Insulinoma b. Glaucoma c. Gastrinoma d. VIPoma A Schwartz’s: p. 1275 110. The most common indication for splenectomy: a. Staging of Hodgkin’s disease b. Hereditary spherocytosis C Schwartz’s: p. 1301 c. Trauma to spleen d. Thalassemia 111. The best time to repair of cleft lip is: a. 10 months b. 10 weeks c. 6 months B Schwartz’s: p. 1797 16 weeks 112. A 36 y/o female complain of an acute pain at the lateral aspect of her breast. A tender firm cord is found on the same site on physical examination. It was diagnosed as a Mondors disease. This lesion is best managed with: a. Radiotherapy b. Anti-inflammatory medications, warm compress and rest of ipsilateral extremity c. Immediate excision of the lesion is needed d. Total mastectomy B Schwartz’s: p. 463 113. Breast lesion that can be treated with close observation with or without tamoxifen: a. Early invasive breast carcinoma c. Ductal carcinoma in situ b. Lobular carcinoma in situ d. Inflammatory carcinoma of the breast B Schwartz’s: p. 481 114. The best screening imaging technique for breast cancer is: a. Magnetic resonance imaging b. Ultrasonography D Schwartz’s: p. 476-477 c. Ductography d. Mammography 115. A 46 y/o female who underwent a total thyroidectomy for stage II papillary carcinoma developed 3 weeks later with Chvostek’s and Trouseau’s sign. This could be due to: a. Recurrence of papillary thyroid carcinoma c. Hypoparathyroidism b. Injury to the cervical sympathetic trunk d. injury to the recurrent laryngeal nerve C Schwartz’s: p. 1429-1448 116. This is considered as a single most important test in the evaluation of patients with thyroid masses a. Thyroid ultrasound c. Fine needle aspiration b. CT and MRI d. Thyroid Scanning C Schwartz’s: p. 1415 117. Lymph node in the posterior triangle of the neck is within what level: a. Level II b. Level IV c. Level V d. Level VI C Schwartz’s: p. 534 118. This form of shock has a low blood pressure, low urine output but has a elevated central venous pressure: a. Vasodilitory shock c. Neurogenic shock b. Obstructive shock d. Traumatic shock B Schwartz’s: p. 95-102 119. 55 y/o male who has been in the ward for the past 4 days and is receiving IV fluid of D5W is complaining on his 5th hospital day of headache, nausea, vomiting, anorexia, body malaise and watery diarrhea. Give the most probable electrolyte imbalance the patient might have: A. Hypernatremia B. Hyperkalemia C. Hyponatremia D. Hypokalemia C Schwartz’s: p.48 120. the most important part of the treatment of severe metabolic acidosis among trauma patient is: A. Correct abnormality with exogenous bicarbonate bicarbonate C. Restore perfusion with volume resuscitation B. High Oxygenation D. Give colloid solution C Schwartz’s: p.50-51 121. the most common primary bone malignancy: A. Osteosarcoma B. Ewing’s Sarcoma C. Ostoblastoma A Schwartz’s: p.1665 D. Chondrosarcoma 122. The most potent stimulant for aldosterone release is: A. ACTH B. rennin C. angiotensin I D. angiotensin II A Schwartz’s: p.9 123. the most biologically active and potent vasoconstrictor is: A. angiotensin II B. endothelins C. epinephrine B Schwartz’s: p.23 D. Serotonin 124. In trauma patients significant reduction in infectious complications were noted in patients given early enteral nutrition as compared to those given who are unfed or given parenteral nutrition except for what type of trauma patients? A. Renal Trauma B. Long bone fractures C. Head trauma D. Liver trauma C Schwartz’s: p.431 125. Branched chain amino acids are used in parenteral nutrition for what specific type of patients? A. renal failure patients C. pulmonary failure patients B. diabetic patients D. hepatic failure patients D Schwartz’s: p.33 126. In what type of parenteral feeding is fat increased to 50%of total calories? A. heart failure formulas C. renal failure formulas B. pulmonary failure formulas D. calorie- defense formulas B Schwartz’s: p.33 127. the appendeceal flora remains constant throughout life with the exception of what bacteria, which appears only in adults? A. Porphromonas gingivalis C. Peptostreptococc i B. Pseudomonas aeruginosa D. Streptococcus Anginosus A Schwartz’s: p.1129 128. The most significant factor associated with both fetal and maternal deaths in pregnant patients with acute appendicitis is A. formation of a phlegmon C. appendeceal tip near the uterus B. appendeceal perforation D. retrocecal appendix B Schwartz’s: p.1129 129. Which of the ff. is true of the superior mesenteric artery syndrome? A. it is often seen in obese individuals B. it usually affects patients over 70 yrs. Old. C. Compression is over the third portion of the duodenum D. it is a surgical emergency C Schwartz’s: p.1027 130. which of the following factors present in a entero-cutaneous fistula increases the possibility of closure of the tract? A. Non-epithelializati on of the tract C. Malnutrition B. High output fistula D. Sepsis A Schwartz’s: p.1037 131. what is the single most important factor in predicting burn related morbidity and mortality? A. size of the burn C. etiology of the burn B. type of the burn D. associated medical condition A Schwartz’s 7th ed., p.228-232/Ans. A 132. what type of shock is caused by the interference of the balance of vasolidator and vasoconstrictor influences to arterioles and venules? A. cardiogenic shock C. septic shock B. hypovolemic shock D. neurogenic shock D Schwartz’s 7th ed., p.113-115/Ans. D 133. A 50 y/o female consulted to you and complains of fatigue, polydipsia, polydipsia, polyuria, nocturia, joint pains and constipation. Her laboratory exams show normal FBS, elevated serum calcium and elevated intact PTH. What would be the most likely diagnosis? A. NIDDM B. Metastatic Breast Cancer C. Hyperthyroidism D. Hyperparathyroidism D Schwartz 7th ed., p. 1434-1439 / Ans. D 134. A 35 y/o male presents with a long standing severe hypertension with associated muscle weakness, headache, polyuria and polydipsia. Blood exams revealed an elevated aldosterone level and elevated serum potassium. What is the most likely diagnosis? Cushing’s Syndrome A. B. Pheochromocytoma C. Conn’s Syndrome D. Waterhouse-Frideric hsen Syndrome C Schwartz 7th ed., p. 1453-1458 / Ans. C 135. A. B. C. D. Which of the following is the most commonly injured organ in blunt abdominal injury? small intestine large intestine pancreas spleen D Schwartz 7th ed., p. 187 / Ans. D 136. In the initial management of an acutely and seriously injured patient, the first and most important emergency measure to be taken care of is: A. splinting of fracture B. control of bleeding C. restoration of blood volume D. ensuring adequate airway D Schwartz 7th ed., p. 156 / Ans. D 137. Diagnostic peritoneal lavage (DPL) remains the most sensitive test available for determining the presence of intraabdominal injury. The result of DPL is considered grossly positive if: A. 1 ml of blood is aspirated B. 5 ml of blood is aspirated C. 8 ml of blood is aspirated D. 15 ml of blood is aspirated D Schwartz 7th ed., p. 167 / Ans. D 138. A. B. C. D. A Schwartz 7th ed., p. 156-157 / Ans. A Which of the following constitute an immediate threat to life because of inadequate ventilation? flail chest hemothorax stabbed wound involving the diaphragm pulmonary hematoma 139. While undergoing exploratory laparotomy for blunt abdominal injury, the patients BP was noted to be 70 palpatory while bleeding was noted coming from the splenic hilum. Appropriate management for the injured spleen should be: A. splenectomy B. splenorrhapy C. debridement and repair D. packing, immediate closure of abdominal lesion and volume replacement A Schwartz 7th ed., p. 193 / Ans. A 140. During the initial resuscitation of a hypovolemic patient secondary to a vehicular accident, external bleeding is best controlled by: A. suturing B. direct finger pressure C. application of tourniquet D. blood transfusion B Schwartz 7th ed., p. 157 / Ans. B 141. A. B Schwartz 7th ed., p. 165 / Ans. B In penetrating neck injuries, zone II is referred to an area between: clavicle and cricoid cartilage B. C. D. cricoid cartilage and angle of mandible above the angle of the mandible below the clavicle 142. In assessing the extent of hepatic injury following blunt injury to the abdomen, bleeding from the liver is best controlled by: A. manual compression of the liver parenchyma B. using figure of 8-suture C. pringle maneuver D. liver resection C Schwartz 7th ed., p. 188-189 / Ans. C 143. Stable patients at risk of urethral injury, manifesting as presence of blood at the matus, necessitates an immediate: A. urethral catheterizat ion B. urethrography C. cystoscopy D. KUB-IVP B Schwartz 7th ed., p. 161-162 / Ans. B 144. Type of healing in which the wound is allowed to heal by granulation tissue formation and contraction: A. primary intention B. secondary intention C. tertiary intention D. quaternary intention B Schwartz 7th ed., p. 234 / Ans. B 145. Partial thickness wound such as seen in superficial second degree burns heal by which of the following process: A. wound contraction C. granulation tissue formation B. epithelization D. maturation and remodelling B Schwartz 8th ed., p. 228 / Ans. B 146. Equilibrium between collagen synthesis and collagen degradation occurred during the stage of: A. hemostasis and inflammation C. matrix synthesis B. proliferative stage D. Remodelling D Schwartz 8th ed., p. 228 / Ans. D 147. Delayed primary wound closure is indicated in which of the following type of wound: A. diabetic ulcer B. decubitus ulcer C. contaminated traumatic wound D. surgical incision following simple appendectomy C Schwartz 7th ed., p. 264 / Ans. C 148. Cytokines that are produced by one cell and affect and adjacent cell such as transforming growth factor beta (TGF-B) are called: A. endocrine factors C. autocrine factors B. paracrine factors D. intacrine factors B Schwartz 7th ed., p. 266 / Ans. B 149. The major component of the extracellular matrix that provides strength, support and structure of all soft tissues, tendons, ligaments and bones is: A. collagen C. fibronectin B. elastin D. hyaluronic acid A Schwartz 7th ed., p. 270-271 / Ans. A 150. The genetic disorder arising from mutations in the genes for the type I collagen causing increase propensity for the bones to break under minimal stress is: A. ehlers-danlos syndrome C. marfan’s syndrome B. osteogenesis imperfecta D. epidermolysis bullosa B Schwartz 7th ed., p. 278 / Ans. B 151. Lesion of the gastrointestinal tract that result in complete regeneration and recovery: A. peptic ulcer disease C. radiation colitis B. crohns disease D. gastric erosions D Schwartz 7th ed., p. 280-281 / Ans. D 152. Skin lesions that extend beyond the boundaries of the original wound, do not regress with time and recur after excision: A. hypertrophic scars C. diabetic ulcers B. keloids D. decubitus ulcers B Schwartz 7th ed., p. 281-282 / Ans. B 153. Kehr’s sign is a classic example of: A. unreferred visceral pain B. referred visceral pain D Schwartz 7th ed., p. 264 / Ans. D 154. The bleeding in Mallory-Weiss syndrome is secondary to: A. penetration of ulcer affecting the gastroduodenal artery B. linear tears of the gastroesophageal junction C. gastric erosions D. hematobilia C. unreferred parietal pain D. referred parietal pain B Schwartz 7th ed., p. 1062-1063 / Ans. B 155. The most common cause of intestinal obstruction for all age group combined is: A. strangulated hernis B. volvulus C. adhesive bands D. neoplasm C 156. The most obvious route of fluid and electrolyte loss in patients with intestinal obstruction from ileocecal TB is through; a. the edematous bowel wall b. the intestinal lumen c. vomiting d. the peritoneal cavity C. vomiting Schwartz 7th ed., p. 1056/Ans. C 157. Occlusion of the blood supply to a segment of bowel in addition to obstruction of the lumen is referred to as; a. complete intestinal obstruction b. closed-loop obstruction c. strangulated obstructio d. ileus C. strangulated obstruction Schwartz 7th ed., p. 1057/Ans. C 158. Which of the following diagnostic modalities consistently localizes the site of bleeding in cases of lower GI bleeding; a. Colonoscopy b. CT scan c. Angiography d. Barium enema C. Angiography Schwartz 7th ed., p. 1066/Ans. C 159. The most common cause of massive lower GI bleeding is; a. colonic malignancy b. diverticulosis c. adenomatous polyps d. ulcerative colitis B. diverticulosi diverticulosiss Schwartz 7th ed., p. 1065/Ans. B 160. A 50 y/o male with history of alcoholism was admitted because of history of progressive jaundice, associated with tea-colored urine, acholic stools, weight loss and mild epigastric pain. The simplest and most non-invasve method in the diagnosis of this patient is; a. percutaneous transhepatic cholangiography (PTC) b. ERCP c. Ultrasonography d. HIDA scan C. Ultrasonography Schwartz 7th ed., p. 1071/Ans. C 161. Relative to the ileum, the jejunum has; a. more fatty mesentery b. less prominent plicae circularis c. longer vasa recta d. lesser in diameter C. longer vasa recta Schwartz 8th ed., p. 1018/Ans. C 162. The most common malignant neoplasm of the small bowel is; a. Adenocarcinoma b. carcinoids c. sarcomas d. lymphomas A. Adenocarcinoma Schwartz 7th ed., p. 1242/Ans. A 163. The presence of a Meckels diverticulum in a hernial sac is called; a. spingelian hernia b. femoral hernia c. littre's hernia d. richter's hernia C. littre's hernia Schwartz 7th ed., p. 1249/Ans. C. 164. Rovsing's sign is elicited by; a. pain in the right lower quadrant when palpatory pressure is exerted in the left lower quadrant b. pain in the right lower quadrant when placatory pressure is exerted directly over the point of tenderness c. flexion-extension of the right lower extremity d. asking the patient to cough A. pain in the right lower quadrant when palpatory pressure is exerted in the left lower quadrant Schwartz 7th ed., p. 1385/Ans. A 165. In children with history of URTI 3 days prior to development of RLQ pain, the differential diagnosis most often confused with appendicitis is; a. Meckel's diverticulitis b. perforated peptic ulcer c. acute gastroenteritis d. acute mesenteric adenitis D. acute mesenteric adenitis Schwartz 7th ed., p. 1387/Ans. D 166. The recommended treatment for patients with adenocarcinoma of the appendix is; a. Appendectomy b. right hemicolectomy c. total colectomy d. subtotal colectomy B. right hemicolectom hemicolectomy y Schwartz 7th ed., p. 1392/Ans. B 167. The precipitating factor in secondary omental torsion is; a. bifid omentum b. accessory omentum c. obesity d. foci of intraabdominal inflammation D. foci of intraabdominal inflammation inflammation Schwartz 7th ed., p. 1556/Ans. D 168. The most common solid tumor of the omentum is; a. Lymphoma b. liposarcoma c. leiomyosarcoma d. metastatic carcinoma D. metastatic carcinoma Schwartz 7th ed., p. 1558/Ans. D 169. The most common anaerobic bacteria that is isolated in intraabdominal infection is: a. clostridium species b. peptococcus species c. bacteriodes species d. Fusobacterium species C. bacteriodes species th Schwartz 7  ed., p, 1524-1525 / Ans. C 170. A 45 year old female came in because of right upper quadrant pain, colicky aggravated by fatty food intake 4 hours ptc. Physical exam showed a palpable mass at the right subcostal margin, non-movable, tender, midinspiratory arrest was illicited while the palpating hand was on the right subcostal area. The most likely condition is/are: a. acute appendicitis b. acute cholecystitis c. perforated peptic ulcer d. acute ascending cholangitis B. acute cholecystitis cholecystitis Schwartz / Ans. B 171. The laboratory examination of a 45 year old female came in with the following results: SGPT was elevated, direct bilirubin increased, alkaline phosphatase increased, protime prolonged. UTZ showed a hyperechoic mass at the neck of the gallbladder with posterior shadowing. The most likely diagnosis is: a. acute appendicitis b. acute infectious hepatitis c. acute calculous cholecystitis d. Ascending cholangitis C. acute calculous cholecystitis Schwartz / Ans. C 172. The treatment of choice for the above condition ( no.4 ) is: a. appendectomy b. medical management c. cholecystectomy cholecystectomy d. ERCP C. cholecystectomy Schwartz / Ans. C 173. A patient s/p cholecystectomy and cbde with t-tube choledochostomy showed a distal cbd stone on ttube cholangiography. Management would be: a. daily irrigation with NSS b. ERCP c. choledococopy and basket extraction of the stone d. repeat explore lap and cbde C. choledococopy and basket extraction of the stone Schwartz / Ans. C 174. A 25 year old male came in with chronic draining lesion at the perianal area of 2 years duration. Physical exam showed a draining lesion about 4 cm from the anal verge at the 3:00 position. The internal opening is probably: a. anterior midline b. posterior midline c. radially directly opposite d. variable B. posterior midline Schwartz / Ans. B 175. 30 year old female came in because of painful perianal mass 2 days ptc. Physical exam showed a tender mass at the right lateral position w/c precludes rectal exam with a bulging tender mass adjacent to it. The most likely condition is: a. thrombosed external hemorhoids b. prolapsed internal hemorhoid c. perianal abscess d. rectal polyp C. perianal abscess Schwartz / Ans. C 176. A 50 year old male came in because of loss of weight about 30% of his previous body weight. Associated with on and off colicky abdominal pain. He also noticed that his stools have diminished in calibre just like a goat stool. The single most important examination would be: C. colonoscopy w/ biopsy Schwartz / Ans. C a. fecalysis b. Ba enema c. colonoscopy w/ biopsy d. CT scan 177. A 26 year old female came in with a palpable breast mass noted 2 weeks ptc. The mass is about 2.5 cm well delineated border, movable, non-tender at the right upper outer quadrant, no familial history of ca. The most likely diagnosis would be: a. Fibroadenoma b. fibrocystic disease c. breast ca d. galactocoele A. Fibroadenoma Schwartz / Ans. A 178. Seromas are fluid collection coming from the: a. Plasma b. Interstilium c. Lymph d. Capillary oozing C. Lymph 179. A patient was discharged 3 days after appendectomy (suppurative). After 5 days, he came back at the OPD for follow up complaining of tenderness at the operative site. The incision site was nonerythematous. What is your diagnosis? a. Hyperesthesia b. Surgical site infection c. Foreign body reaction to suture d. Urinary Tract infection B. Surgical site infection 180. A patient was operated on for acute cholecystitis. After 24h postop, the patient developed fever. What is the most likely cause of this fever? a. Cathether sepsis b. Drug-related conditions c. Aspiration pneumonia d. Atelectasis D. Atelectasis 181. The basic protein in poor wound healing regardless of the underlying factor is: a. Poor local hemostasis b. Anemia c. Low O2 tension d. Impaired inflammatory response C. Low O2 tension 182. Ileus following abdominal surgery is expected to last for at least how many days? a. 3 b. 4 c. 5 d. 6 C. 5 3 183. For every degree rise in the temperature, the insensible loss in cm  per day is approximately: a. 100 b. 150 c. 200 d. 250 D. 250 184. Which ion if altered determines the shift of fluid from one compartment to another? a. Sodium b. Potassium c. Chloride d. Bicarbonate A. Sodium 185. Which of the following cytokines may help control keloids and hypertrophic scars? a. PDGF b. EGF c.TNF d. TGB-B d. TGB-B 186. Which part of the GIT provides strength in the anastomosis? a. Serosa b. Muscularis c. Submucosa d. Mucosa C. Submucosa 187. Which amino acid provide energy source in the GIT? a. Alanine b. Valine C. Glutamine c. Glutamine d. Tryptopan rd 188. A 54 y/o male patient with 3  degree burn approximately 40% TBSA developed abdominal pain on the 4 hospital day. What is the least likely cause of the abdominal pain? a. Acalculous cholecystitis b. Acute pancreatitis c. Superior mesenteric artery syndrome d. Renal colic th D. Renal colic 189. Convulsions if present during resuscitative phase of burn injury may be due to a. Hypoxemia b. Hyponatremia c. Infection d. Hypokalemia B. Hyponatremia 190. The offending organism in surgical site infection is: a. Staph aureus b. Pseudomonas c. E coli d. Bacteroides A. Staph aureus 191. Earliest manifestations of the catheter related complication following parenteral nutrition a. Tachycardia Tachycardia b. Fever c. Glucose intolerance d. Changes in sensorium C. Glucose intolerance 192. Which of the following statements is not a sound principle in the fluid and electrolyte therapy post operatively? a. Thorough evaluation of the pre and intra-op fluid status b. Urine volume is replaced on a milliliter to milliliter basis c. It is not necessary to give potassium with in the first 24 hours d. Insensible water loss is considered in the fluid therapy B. Urine volume is replaced on a milliliter to milliliter basis 193. The average potassium (in meq/l) content of the bile per day is a. 5 b. 10 c. 15 d. 18 B. 10 194. Which cranial nerve is involved in the act of swallowing a. 11 b. 10 c. 7 d. 12 A. 11 195. Primary reason for staging esophageal cancer is a. Determine its resectability b. Prognostication c. To assess whether the procedure is for cure or palliation d. Whether pre-op chemotherapy is indicated A. Determine its resectability 196. Most common benign esophageal tumor is a. Leiomyomas b. Fibromas c. Myomas d. Fibromyomas A. Leiomyomas 197. Blood supply of the lesser curvature is a. Left gastric b. Right gastric c. Gastroepiploic d. Splenic B. Right gastric 198. Manifestation of acute gastric dilatation a. Pallor b. Rapid respiration c. Changes in sensorium d. Hypotension D. Hypotension 199. A patient underwent upper gastrointestinal endoscopy for gastric outlet obstruction. The endoscopist noted a lesion at the antrum and took biopsy. It turned out to be gastric cancer. What type of gastric cancer has B. Polypoid favorable prognosis? a. Superficial spreading b. Polypoid c. Ulcerative d. Linitis plastic 200. A patient diagnosed to have perforated peptic ulcer disease has his symptoms 8 hours prior to admission. He was prepared for surgery. Which of the following best describes the required operation? a. repair of perforation and selective vagotomy b. repair of perforation and truncal vagotomy c. suture of perforation d. omental patch D. omental patch 201. After truncal vagotomy for peptic ulcer disease, the patient may develop the following except: a. gastric stasis b. megaloblastic anemia c. diarrhea d. gallstone formation B. megaloblastic anemia 202. Most common neoplasm of the stomach a. adenocarcinoma b. lymphoma c. leiomyosarcoma d. squamous cell carcinoma A. adenocarcinoma 203. A 77 y/o, M, chronic smoker on NSAIDs for arthritis presents with an acute abdomen and penumoperitoneum. What could be the most likely cause of this? a. severe abdominal pain due to chronic gastritis b. acute pancreatitis c. acute cholecystitis cholecystitis with cholangitis cholangitis d. perforated peptic ulcer disease A. severe abdominal pain due to chronic gastritis 204. Increased venous pressure, decreased pulse pressure and decreased heart sounds are pathognomonic of: a. acute MI b. pneumothorax c. cardiac tamponade d. aneurysm of the arch of aorta C. cardiac tamponade 205. “Bird’s beak” deformity is a radiologic sign of  a. sigmoid volvulus b. ileoascending intussuception c. sigmoid carcinoma d. cardiac tamponade A. sigmoid volvulus 206. A 32 y/o M was brought to the ER because of blunt abdominal trauma due to fall. The patient had hematoma and abrasions on his RUQ and Rt flank. Your resident assessed that the patient has surgical abdomen and needs immediate surgery. He requested one-shot intravenous pyelography en route to OR. The reason for such, a. find out extravasation of urine b. localize site of obstruction c. determine function of either kidneys d. quantify volume of urine output C. determine function of either kidneys 207. A 23 y/o female, medical student, was brought to the ER because of VA. Patient was GCS 13-14 with blunt abdominal trauma following a motor vehicle accident. Diagnostic peritoneal lavage was done and showed negative results. The assessment of negative result signifies: a. no hollow viscus injury b. absent blood in peritoneal cavity c. needle did not reach the peritoneal cavity d. does not rule out abdominal organ injury D. does not rule out abdominal organ injury 208. The most important factor in the management of contaminated wounds is a. broad spectrum antiobiotics b. anti-tetanus prophylaxis c. adequate debridement d. closure without tension C. adequate debridement 209. Whether it’s diverticulization or exclusion, the principle behind these procedures for the duodenal injuries is to a. prevent fistula formation b. to rest or isolate the injured duodenum c. prevent leakage d. contain the inflammation at the RUQ B. to rest or isolate the injured duodenum 210. Sphincterotomy for anal fissures is usually done at the a. anteromidline b. posterolateral c. posteromidline d. anterolateral B. posterolateral 211. Surgical management of familial adenomatous polyposis depends on the a. number of polyps seen b. size of polyps c. histologic grading d. staging A. number of polyps seen 212. Elective colorectal cancer operation is classified as a. clean b. contaminated c. clean contaminated d. dirty C. clean contaminated 213. Which of the following is the most important prognostic determination of survival after treatment of colorectal cancer a. lymph node involvement b. transmural extension c. tumor size d. histologic differentiation D. histologic differentiation 214. A 60 y/o male is admitted to the hospital after passing out large amount of maroon-colored stools. At the ER, he again passed out more bloody stools as well as clots. He is pale and tachycardic, NGT aspirate is bilious. After resuscitation, which of the ff is the most appropriate initial test: a. barium enema b. rigid proctoscopy c. colonoscopy d. RBC tag scan D. RBC tag scan 215. Which is the most important stimulus for triggering endocrine response to injury a. hypovolemia b. afferent nerve stimulation from the injured area c. tissue acidosis d. local wound factors D. local wound factors 216. Which is not true regarding gastrostomy tube feeding? a. can be done endoscopically or open method b. indicated for mentally obtunded patients c. caloric requirement is based on patient’s needs d. preferably delivered in a constant flow D. preferably delivered in a constant flow 217. Pellagra, dermatitis, glossitis, & peripheral paresthesias with spinal cord symptoms are due to deficiency of a. Folic acid b. Vitamin a c. Pantothenic acid d. Zinc A. Folic acid 218. The primary source of energy following trauma is a. glycogen b. lipids c. protein d. carbohydrates A. glycogen 219. Eicosanoids are cell mediators derived from a. glycerol b. cholesterol c. arachidonic acid d. glutamine C. arachidonic acid 220. The reason why approximately one half of breast cancer are located at the upper outer quadrant is a. its lymphatics b. the predilection of cancers superiorly c. the larger the volume of breast tissue d. abundance of blood supply D. abundance of blood supply 221. Which structures demarcates the node levels in breast cancer: a. latissimus dorsi D. pectoralis minor b. pectoralis major c. axillary artery d. pectoralis minor 222. Carcinoma of the breast among the elderly presenting as a bulky, colloid tumor. a. Mucinous b. Medullary c. Adenoid d. Apocrine A. Mucinous 223. Management of inflammatory breast carcinoma with the best response a. Neoadjuvant chemo + MRM + irradiation b. Classical mastectomy + irradiation c. Chemotherapy + irradiation d. MRM + Hormonal manipulation A. Neoadjuvant chemo + MRM + irradiation 224. Which of the following structures which when encroached by cancer gives rise to the “orange-peel” appearance of the breast? a. Glands of Montgomery b. Ductal ampulla c. Lobules d. Subdermal lymphatics D. Subdermal lymphatics 225. A 34 female patient came at the OPD with 2cm palpable mass, moveable, nontender. No axillary nodes noted. Patient has an elder sibling who died of breast cancer last 2 years ago. What will be your initial diagnostic procedure? a. Needle or open biopsy b. Mammography c. Open biopsy ultrasound of the breast d. Magnetic resonance imaging A. Needle or open biopsy 226. Collagen synthesis is at its peak at around how many days? a. 1-2 b. 3-5 c. 5-7 d. 10-14 C. 5-7 227. Formation of collagen fibers by cross-linking is enhanced by lysyl oxidase together with a. magnesium b. copper c. zinc d. chromium B. copper 228. Which of the following steroids useful in the treatment of scars and keloids a. Betamethasone b. Triamcinolone c. Prednisolone d. Cortisone B. Triamcinolone 229. Most common cause of hospital acquired infections are coming from the a. Respiratory tract b. Intraabdominal c. Urinary tract d. Blood C. Urinary tract 230. 45 male came in to the ER with a 3cm lacerated wound at the left leg after he was bitten by a pig. He has received tetanus toxoid and tetanus immunoglobulin 10 ½ years ago. What would be the appropriate management at the ER? a. Toxoid +tetanus+ antibiotics b. Tetanus immunoglobulin only c. Debridement, leave wound open + tetanus immunoglobulin + toxoid+ antibiotics d. Tt + ATS + Cloxacillin C. Debridement, leave wound open + tetanus immunoglobulin immunoglobulin + toxoid+ antibiotics 231. Superior vena caval obstruction is commonly caused by a. Fibrosing mediastinitis b. Bronchogenic carcinoma c. Caval thrombosis d. Aortic aneurysm B. Bronchogenic carcinoma 232. The average length of the adult trachea is: a. 8 cm b. 9 cm c. 10 cm D. 11 cm d. 11 cm 233. Squamous cell carcinoma of the lung is commonly caused by a. hypercalcemia b. hyperkalemia c. hyperglycemia hyperglycemia d. hypomagnesemia A. hypercalcemia 234. The initial diagnostic tool in assessing biliary ductal obstruction a. Alkaline phosphatase b. Ultrasonogrophy c. ERCP d. CT Scan A. Alkaline phosphatase “Courvoisier’s sign” suggest 235. Progressive painless  jaundice with positive “Courvoisier’s a. Choledocholithiasis b. Hydrops of gall bladder c. Pancreatic head or peri ampullary neoplasm d. Choledochal cyst C. Pancreatic head or peri ampullary neoplasm 236. The initial diagnostic procedure in patients presenting with lower GI bleed a. NGT intubation b. Proctosigmoidoscopy c. Barium enema d. Colonoscopy B. Proctosigmoidoscopy Proctosigmoidoscopy 237. The main parameter for determining the clinical stage of head and neck tumor is a. Size of primary tumor b. Lymph node involvement c. Metastasis d. Functional loss B. Lymph node involvement 238. Malignancy of the tongue usually occurs at the a. Tip b. Midlateral c. Central d. Posterior B. Midlateral 239. The second most frequent benign salivary gland neoplasm is a. Pleomorphic adenoma b. Oxyphilic adenoma c. Papillary cystadenoma lymphomatosum d. Sialadenoma C. Papillary cystadenoma lymphomatosum 240. The acidosis in hypovolemic shock is due to a. Phosphate b. Lactic acid c. Carbonic acid d. HCL B. Lactic acid 241. Substance associated with increased incidence of mesothelioma a. Smoking b. Silica c. Asbestos d. Lead C. Asbestos 242. Prostatic carcinoma usually metastasizes to the a. Liver b. Kidneys c. Bladder d. Bone D. Bone 243. The earliest visible evidence of neoplastic transformation is a. Hyperplasia b. Metaplasia c. Dysplasia d. Angiogenesis C. Dysplasia 244. Epstein-barr virus is associated with the following tumors, except a. Burkitts lymphoma b. Nasopharyngeal carcinoma c. Hodgkin’s d. Hepatocellular carcinoma D. Hepatocellular carcinoma 245. A normal to low CVP that does not rise with rapid administration of crystalloids indicates: a. Inadequate rate of infusion b. Improper placement of CVP c. Continuing hypovolemia d. Improper choice of crytalloids A. Inadequate rate of infusion 246. Recurrent laryngeal nerve is preserved in thyroid surgery. What is false with regards to this nerve: a. Injury to both nerves cause upper airway obstruction b. Injury to the ipsilateral nerve cause disability of phonation c. It is a branch of cranial nerve x d. 64% of the recurrent is seen along the tracheosophageal groove B. Injury to the ipsilateral nerve cause disability of phonation 247. Among the types of thyroid cancer, which one is the most aggressive? a. Follicular b. Papillary c. Hurthle d. Anaplastic D. Anaplastic 248. A 45yo female patient underwent total thyroidectomy for follicular ca. 24 H after surgery, the patient developed numbness around the lips and finger. What could have caused this? a. Transection of recurrent nerve b. Ischemia to parathyroids c. Hematoma d. Transection of superior laryngeal nerve B. Ischemia to parathyroids 249. True regarding hernia: a. Most common type of hernia in females is direct hernia b. Strangulation is more common in indirect than in femoral hernia c. Tension-free repair is best achieved with bassini technique d. Repair of umbilical hernia is indicated in infants with hernia > 2cm D. Repair of umbilical hernia is indicated in infants with hernia > 2cm 250. A 23yo male patient came in at ER with irreducible inguinal hernia of >16 hours duration. Patient complained of abdominal pain with BP of 100/70, CR 120, & RR 30. The appropriate management is: a. Observe for progression of pain b. Sedate patient and reduce the hernia c. Apply ice pack over the hernia d. Schedule patient for surgery D. Schedule patient for surgery 251. At the ER, a patient complained of renal colic. The following are the minimal laboratory evaluation you would request except: a. CBC and electrolytes b. IVP c. Bun and creatinine d. Urinalysis A. CBC and electrolytes 252. In the medical treatment of BPH, which among the drugs acts by reducing the intraprostatic dihydrotestosterone levels without lowering the plasma testosterone level? a. Doxazosin b. Finasteride c. Terazosin d. Tamsulosin B. Finasteride 253. A 48hour old baby boy was brought to the ER due to vomiting of bilious vomitus, abdominal distension, and failure to pass out meconium. What is the most likely cause? a. Tracheo-esophageal atresia b. Ileal atresia c. Pyloric stenosis d. Rectal mass B. Ileal atresia 254. A 28 kg child with ruptured appendicitis has the following laboratory results: BP=90/60; PR=110; RR=25; Na=125; Cl=95; K=3; ph=7.35; pCO2=35; pO2=100; HCO3=24. Which of the following is TRUE? a. anion gap is increased b. uncompensated metabolic acidosis is present c. actual Na deficit is 250 mEq d. the ideal intravenous fluid to infuse is D50.3% NaCl C. actual Na deficit is 250 mEq 255. Interpret the above ABG results: a. compensated respiratory alkalosis b. uncompensated metabolic acidosis c. compensated metabolic acidosis d. uncompensated metabolic alkalosis C. compensated metabolic acidosis 256. In the above patient, which of the following is TRUE? a. plasma volume is 5,600 cc b. initial fluid resuscitation is 560 cc given as fast drip c. decrease in Na is primarily due to renal losses d. decrease in Na is delusional in nature B. initial fluid resuscitation is 560 cc given as fast drip 257. In the same patient, which abnormality is he liable to manifest? a. increased intracranial pressure b. dry sticky mucous membrane c. shift of oxygen-hemoglobin dissociation curve to the left d. ventricular arrhythmias A. increased intracranial pressure 258. Extracellular deficit results in: a. increased sodium b. increased hematocrit c. decreased BUN d. decreased creatinine B. increased haematocrit 259. A 70-kg is admitted because of abdominal colic followed by vomiting of previously ingested food. Initial assessment is an Intestinal Obstruction probably from adhesions. On examination, the patient has weak and rapid pulse, depressed eyeballs, and dry tongue and mucous membranes. The most likely fluid state of the patient is: a. hypotonic dehydration b. isotonic dehydration c. hypertonic dehydration d. hypernatremia B. isotonic dehydration 260. TRUE statements regarding fluid and electrolyte losses in this patient: a. vomiting accounts for an insignificant amount of the volume loss b. can go into hypovolemic shock anytime c. sequestration of fluid in the urinary system accounts for majority of the losses d. compensatory fluid shift from other compartments is sufficient to maintain homeostasis B. can go into hypovolemic shock anytime 261. Fluid replacement in this patient should begin with: a. blood b. colloids c. crystalloids crystalloids d. plasma C. crystalloids 262. Which of the following is an expected cellular change/s in hypovolemic shock? a. intracellular gain of Na+ and H2O b. depletion of cellular ATP c. extracellular increase of K+ d. increase in transmembrane potential D. increase in transmembrane potential 263. A patient has a blood pressure of 70/50 mmHg and a serum lactate level of 30-mg/100 ml (normal: 6 to 16). His cardiac output is 1.9 L/min, and his central venous pressure is 2 cmH2O. The most likely diagnosis is: a. congestive heart failure b. hypovolemic shock c. cardiac tamponade d. pulmonary embolus B. hypovolemic shock 264. During compensation for hypovolemic shock, blood flow to which of the following area/s is under sympathetic nervous system control? a. brain b. heart c. small bowel d. liver C. small bowel 265. This hemodynamic monitor is valuable to differentiate a pericardial tamponade from hypovolemia: a. CVP b. Arterial catheter c. pulmonary catheter d. capnograph A. CVP 266. In a multiply injured patient with acute blood loss, adequate preload to the heart is maintained initially by the: a. increase in systematic vascular resistance b. development of tachycardia c. hormonal effects of angiotensin d. hormonal effects of antidiuretic hormone A. increase in systematic vascular resistance 267. An example of neurogenic shock. A. Convulsion B. Syncope C. Carpopedal spasm D. Involuntary fine tremors B 268. Coagulation factors test by prothrombin time (PT) A. VII B. XXII C. VIII D. XI A 269. The initial hemostatic response to injury is A. Platelet formation B. Hemolysis C. Vascular constriction D. Initiation of coagulation cascade C 270. All are major events in the hemostatic process: A. Vascular dilatation B. Platelet plug formation C. Fibrin deposition D. Hemolysis B 271. The following are suggestive of DIC: A. Platelet counts of 250,000/cu. Mm B. Increased fibrin split products C. Normal fibrinogen level D. Hypofribrinogenemia B 272. Injury to the intima of blood vessels exposes the subendothelial collagen which initiates: A. Fibrinolysis B. Platelet aggregation C. Release of cyclic AMP D. Vasoconstriction B 273. A 77-year old man is schedules to undergo sigmoid colectomy. He denies any history of prolonged bleeding. Preoperative evaluation of hemostasis should include: A. No screening tests B. Prothrombin time (PT) and partial thromboplastin time (PTT) C. Platelet count, blood smear, and PTT D. Platelet count, PT, and PTT D 274. The total caloric requirement of a malnourished patient is calculated using his: A. Actual weight B. 75% of ideal body weight C. 50% of ideal body weight D. Ideal body weight A 275. In prescribing the nutrition therapy, micronutrients are given at what % of RDA: A. 100 B. 80 C. 60 D. 50 A 276. Nitrogen excretion is less in A. Starvation B. Sepsis C. Severe burns D. Elective surgery A 277. At the initial stages of shock it is normally expected to have a A. Normal blood pressure B. Hypertension C. Cold extremities D. Hypothermia A 278. A 38-year old male had abdominal exploration for multiple gunshot wounds. He is febrile T 38.5 C on the 2nd post-op day. HR is 95 bpm, BP 100/80 mm Hg, WBC count 13,000. The patient is hooked to a ventilator. The patient is developing what condition: A. SIRS B. Sepsis C. Severe sepsis A D. Septic shock th 279. On the 5  day post-op the patient in addition to the above findings now shows erythema and draining pus from the abdominal incision site. The patient now is developing: A. SIRS B. Sepsis C. Severe sepsis D. Septic shock th B 280. On the 10  day post-op, the patient is persistently, febrile with increase in WBC count (16,000) and this time with oliguria that is unresponsive to fluid resuscitation. The patient now is developing: A. SIRS B. Sepsis C. Severe sepsis D. Septic shock D 281. The above patient was given pressor agents to improve his condition, despite this he remains oliguric. The patient now has: A. SIRS B. Sepsis C. Severe sepsis D. Septic shock D 282. Immediately after mastectomy your patient asks if any of the following promotes wound healing, which one will you advise your patient: A. Vitamin C B. Zinc supplementation C. Vitamin B D. Carbohydrate rich foods A 283. Severe cases of hidradenitis suppurativa in the groin area are best managed by excision of the involved area and: A. Closure by secondary intention B. Delayed primary closure C. Primary closure D. Partial thickness skin grafts A 284. An elderly cancer patient fell and sustained a deep lacerated wound over the right forehead about 9 cm length. If wound infection later develops in this patient. The major cause of impaired wound healing is: A. Anemia B. Local wound infection C. DM D. All of the above 285. What technique of wound closure is recommended in a patient with ruptured appendix with spreading peritonitis? A. Primary B. Delayed primary C. Secondary D. Tertiary B 286. If the lacerated wound is sutured closed, this healing is known as: A. Primary B. Secondary C. Tertiary D. Delayed primary A 287. Wound healing in this this patient may be impaired because of his: A. Age B. Depth of the wound C. Type of wound D. Cancer A 288. Infection in the above wound is partly controlled by the presence of what cells: A. Macrophages B. Fibroblast C. Endothelial cells D. T lymphocytes A 289. Important activities of macrophages during wound healing A. Wound debridement B. Arginine synthesis C. Activation of coagulation cascade A D. Fibrin clot A 290. The ABC’s of resuscitation includes the following: A. Establish adequate airway B. Bleeding should be controlled by applying tourniquet C. Circulation should be maintained by repaid infusion of blood D. Oxygen should be delivered if necessary th 291. A 25 year old male is brought to the ER after he sustained a stab wound to the 5   intercostal space. PE: conscious, stretcher-borne, vitals are as follows: blood pressure 90/50 mm Hg, pulse 110/min and respiratory rate 30/min. Which of the following statements is true: A. Primary survey and initial resuscitation should be instituted one after the other B. A complete detailed physical exam should be performed so as not to miss any injury C. Large-bore IV lines should inserted D. Blood for CBC and x-matching should be drawn C 292. An 8 year old boy has a closed angulated deformity of his right forearm. VS: BP=99mmHg systolic and PR=110. Identify the most emergent problem of the victim. Write any of the following: A. Airway or Cervical Spine B. Breathing C. Circulation D. Disability or Neurologic Injury D 293. A 7 year old boy is gasping for breath. He can speak clearly, but complains of chest pain and an inability to get his breath. He is becoming increasingly anxious. VS: BP=110 mmHg systolic and PR=110. Identify the most emergent problem of the victim. Write any of the following: A. Airway or Cervical Spine B. Breathing C. Circulation D. Disability or Neurologic Injury B 294. A 25 year old man is unconscious, withdraws from painful stimuli, eyes are closed but they open in response to pain, and he is making unintelligible sounds. His pupils are equal, and both react sluggishly to light. His Glasgow coma scale score is. A. 5 B. 8 C. 11 D. 14 B 295. After control of this patient’s airway is achieved, the first diagnostic study to obtain is: A. x-rays of the skull B. CT scan of the head C. x-rays of the cervical spine D. carotid angiography C 296. A 19 year old woman presents with a non-bleeding stab wound, 1 cm long, in the anterior triangle of her neck, 3 cm above her left clavicle. Examination of the neck reveals that the wound has penetrated the platysma muscle and that the left carotid pulse is normal; there is no hematoma or bruit. Appropriate management of this patient: A. immediate tracheostomy B. nasogastric intubation C. chest x-rays D. formal neck exploration under general anesthesia D 297. Following an automobile accident, a 30 year old woman is discovered to have posterior pelvic fracture. Hypotension and tachycardia respond marginally to volume replacement. Once it is evident that her major problem is free intraperitoneal bleeding and a pelvic hematom in association with the fracture, appropriate management would be: A. apply antishock trousers and exploratory laparatomy B. arterial embolizataion of the pelvic vessels C. exploratory laparatomy and possible bilateral internal iliac artery ligation D. external pelvic fixation to stabilize the pelvis C 298. Large amount of free intraperitoneal blood is seen in a patient with penetrating abdominal injury after an abdominal ultrasound, the next best step is to: A. apply antishock trousers and exploratory laparatomy B. arterial embolizataion of the pelvic vessels C. exploratory laparatomy D. external pelvic fixation to stabilize the pelvis C 299. A 6 year old is having episodes of watery diarrhea for the past three days. She is lethargic, irritable and restless. The heartbeat is rapid but the pulses are weak. The child should: A. stay home and call the paediatrician for advice C B. C. D. stay home and have a cold batch proceed to the nearest hospital emergency room get a glass of cold fruit juice to drink 300. Over- enthusiastic fluid resuscitation should be avoided in this child because of A. potential liver failure B. Acute myocardial infarction C. venous thrombosis and rupture D. potential cerebral injury D 301. A study on ruptured appendicitis will be conducted in a tertiary university hospital that includes all age groups and all genders on the rate of SSI between primary vs. delayed primary closure. What is the possible confounder in this study? A. all age groups B. ruptured appendicitis C. SSI D. primary vs. delayed primary A 302. The most common type of inguinal hernia in both males and females is: A. indirect B. direct C. femoral D. mixed A 303. The Cooper’s ligament repair for direct or femoral hernia is popularized by: A. Bassini B. McVay C. Halsted D. Ferguson B 304. A 22 year old female consulted a physician because of a 2x2x3 cms, movable well circumscribed, nontender mass, noted accidentally while taking a bath. Physical examination was consistent with the findings above. Most like diagnosis is: A. Breast Ca B. Fibrocystic change C. Fibroadenoma D. Phylloides tumor C 305. The next thing to do on the above patient is: A. reassurance and observation B. ask for mammogram C. Partial mastectomy D. Hormonal reatment A 306. Ivory an 18 year old female consulted because of breast tenderness becoming more severe just before menses. PE showed asymmetric nodularities on the upper outer quadrant of both breasts with no dominant mass. Menstrual cycle is monthly and regular. Most likely diagnosis is: A. Sclerosing adenosis B. Normal premenstrual tension C. Fibrocystic change D. Breast Ca C 307. Sarah Jane a 35 year old female consulted because of an intermittent bloody nipple discharge on the right breast. PPE-no mass noted on the right breat. Most likely diagnosis is: A. papillary cancer B. mammary duct ectasia C. bleeding galactocoele D. intraductalpapilloma D 308. Chemotherapy in breast carcinoma is: A. given to those with at least 4 positive xillary metastasis B. usually a combination of multiple agents C. free of any side effects D. used to control local recurrence B 309. Breast malignancy that is known to have the highest incidence of bilaterality and multicentricity: A. Paget’s disease of the breast B. lobular carcinoma C. Papillary carcinoma D. Medullary carcinoma 310. The next appropriate thing to do in managing the above case will be: A. Local wound exploration B. CT scan D A C. D. Immediate surgery Triple angiography of the abdomen 311. Fifteen minutes later the BP became 60 palpatory, PR-120/min accompanied by abdominal distension. You will recommend: A. CT scan B. proceed with local wound exploration C. MRI D. Immediate surgery D 312. The most commonly injured intraabdominal organ following a blunt abdominal trauma: A. Pancreas B. Spleen C. small intestine D. colon B 313. A 55 y/o male, smoker underwent FNAB for a 4 cm preauricular mass. Results showed (+) for malignant cells, patient is most probably suffering from: A. Pleomorphic adenoma B. Follicular carcinoma C. mucoepidermoid carcinoma D. papillary carcinoma C 314. The appropriate treatment for a benight parotid neoplasm is: A. “shelling out” of the tumor B. superficial parotidectomy C. total parotidectomy D. radiation therapy B 315. Most common histology of head-neck cancers is A. Adenocarcinoma B. Papillary carcinoma C. squamous cell carcinoma D. Adenosquamous carcinoma C 316. The most common causative agent being implicated in head-neck cancers is A. UV radiation B. Tobacco C. nitrates D. alcohol B 317. The site of injury corresponds to which zone of the neck? A. Zone 1 B. Zone 2 C. Zone 3 D. Zone 4 A 318. The diagnosis of the above is: A. Primary hyperparathyroidism B. Secondary Hyperparathyroidism C. Tertiary Hyperparathyroidism D. Parathyroid Carcinoma B 319. The course of papillary thyroid cancer is best described by which of the following statements? A. Metastases are rare; local growth is rapid B. Local invasion and metastases almost never occur C. Metastases frequently occur to cervical lymph nodes, distant metastases and local invasion are rare D. Rapid widespread metastatic involvement of the liver, lungs and bone results in a 10% 5 yr survival rate C 320. The most important prognosticating factor in thyroid cancer is: A. tumor size B. patient’s age C. lymph node metastases D. Histologic grade B 321. 43 years old female present with diffuse enlargement of the thyroid gland accompanied by palpitation, heat intolerance and weight loss. She is most probably suffering from A. Frey’s syndrome B. Hypothyroidism C. hypocalcemia D. hypokalemic acidosis C 322. A 50-year old male has just undergone a total thyroidectomy for follicular cancer of the thyroid. On the third hour post-op, he experienced circumoral numbness. He is probably suffering from A. Frey’s syndrome B. hypothyroidism C. hypocalcemia D. hypokalemic acidosis C A 16 year old high school student came to you with a complaint that her friends took notice of an anterior neck enlargement since she started on her weight reduction program. You were indecisive as to whether there really was a thyromegaly. The rest of the P.E. were normal C 323. Which would be most helpful in determining whether there really is a thyromegalyor none: A. a second opinion from an endocrinologist B. MRI of the neck C. Thyroid Scanning D. Serum TSH, T3 and T4 324. A 72 year-old male consults because of change in bowel habits for the last two weeks. On rectal exam, a hard irregular mass was noted at 6 cm from the anal verge. The most likely diagnosis here is: A. Rectal carcinoid B. Rectal carcninoma C. Rectal polyp D. Anal canal carcinoma B 325. The most appropriate initial diagnostic procedure to be done is A. ultrasound of the abdomen B. Proctosigmoidoscopy C. Chest x-ray D. CT scan of the pelvis B 326. Majority of peptic ulcer patients are successfully managed medically or conservatively. Which of the following complications is the most common indication for surgical intervention? A. Perforation B. Massive GI bleeding C. Gastric outlet obstruction D. Malignant degeneration B 327. A 48 year old company manager who has been taking proton pump inhibitor irregularly for the past 2 years because of on and off epigastric discomfort presents in the emergency room with hematemesis and melena. This patient should be: A. Aggressively resuscitated with fluids and measures to localize site of bleeding should be done in preparation for possible surgery B. Explored immediately because bleeding is an indication for surgery C. Give massive doses of anti-ulcer medications D. Watchful waiting for 24 to 48 hours A 328. A 43 year old male patient with on and off epigastric pain during the past 15 months complains of sudden severe epigastric pain later on becoming generalized. An upright chest file taken at the ER shows a strip of lucency underneath the right hemidiaphragm. This patient has: A. Penetrating peptic ulcer B. Acute pancreatitis C. Perforated diverticulitis diverticulitis D. Perforated peptic ulcer D 329. Above patient is best managed by: A. Aggressive medical treatment for PUD B. Laparotomy after resuscitation/IV antibiotics C. Fluid resuscitation, nasogastric decompression and somatostatin D. Emergency ERCP B 330. A 30 y/o male who usually has dripping fresh blood after defecation suddenly had anal pain and a smooth, tender 1cm diameter rounded mass coming out of the anus. This is most likely: A. peri-anal abscess rd B. internal hmorrhoids 3  degree C. anal wart D. thrombosed internal haemorrhoids D 331. A 65 y/o male was referred for surgical consult because of abdominal pain and distention. Findings in the scout film of the abdomen that will be highly suggestive of intestinal obstruction A. air in distal rectum B. pneumatosis intestinales C. luscency beneath the right hemidiaphragm D. step ladder abnormality D 332. A 75 y/o female is being worked up for paralytic ileus. The electrolyte that is usually below normal is: A. potassium B. sodium C. calcium D. magnesium A 333. A significant and independent influence on the prognosis of a patient who had an esophageal resection for cancer is: A. tumor presentation of the esophageal wall B. cell type C. degree of cellular dfferentation D. location of the tumor in the esophagus A 334. Hepatic resection should be consider for localized metastatic spread from which of the following primary site? A. colorectal B. stomach C. lungs D. breast A 335. The standard of treatment for patient with biliary colic and cholecystolithiasis by ultrasound is A. Cholecystectomy B. Cholecystostomy C. Stone dissolution D. Endoscopic Stone removal A 336. In an infected obstructed common bile duct producing cholangitis, which of the following is the best option for drainage in a very poor risk patient? A. cholecystostomy B. operative CBD exploration and T tube choledochostomy C. ERCP and stenting D. MRICP C 337. CT scan finding that is highly indicative of infected pancreatic necrosis: A. edema B. dilated pancreatic duct C. calcifications D. gas bubbles in the retroperitoneum D 338. When gallstone ileus occurs, obstruction is most frequent in: A. duodenum B.  jejunum C. distal ileum D. sigmoid colon C 339. The most common cause of acute cholecystitis is: A. cystic duct obstruction B. E. coli infection C. gall bladder polyp near the neck D. multiple gallstones A 340. The strongest evidence of presence of gallstones by ultrasound is: A. edema and thickening of gallbladder wall B. contracted gallbladder C. dilated extrahepatic bile ducts D. High level echoes with posterior acoustic shadowing D A 45 y/o jaundiced patient who has been diagnosed to have gallbladder and CBD stones by ultrasound 6 months ago is brought to the ER because of abdominal pain, chills, high grade fever and hypotension. PE: icteric sclera, abdomen slightly tender with guarding RUQ. C 341. The patient most likely is having which of the fllowing: A. acute cholecystitis B. empyema of the GB C. acute suppurative obstructive cholangitis D. ruptured GB 342. The problem of patient in the previous number is best managed by: A. administration of oral antibiotics B. careful observation and follow up C. surgical/endoscopic decompression of biliary tree after resuscitation and antibiotics D. give systemic antibiotics and steroids C 343. The most likely diagnosis in a 70 y/o male presenting with significant weight loss accompanied by progressive jaundice, anorexia, pruritus, and tea colored urine. PE showed a palpable nontender gallbladder. A. hepatoma B. gallbladder CA C. pancreatic head CA D. gallbladder empyema C 344. The severity of pancreatitis, particularly in those patients who are not improving after 24hours of medical management can be assessed using: A. angiography B. scout film of the abdomen C. dynamic CT scan D. MRICP 345. Which of the following clinical clues is a reliable symptom of arteriosclerosis obliterans? A. poor hair growth B. deformed toenails C. cold hands and feet D. intermittent claudication D 346. What is the most common symptom of acute aortic dissection? A. syncope B. severe pain C. hemiparesis D. congestive heart failure B 347. This is the BEST procedure for a patient with acute head injury secondary to a vehicular accident: A. CT scan B. angiography C. plain skull x-rays D. electroencephalogram A 348. Abrasion wounds on the face are best treated with: A. wound cleansing and debridement with normal saline B. split thickness skin grafting C. laser surgery D. AOTA are correct A 349. Essential elements in the survival of a skin graft: A. antibiotics B. dry dressing C. vascularity D. nerve supply C 350. Most common cause of UTI in children: A. E. coli B. S. saprophyticus C. Klebsiella, Enterobacter D. Proteus sp. A 351. A massively bleeding posterior duoedenal ulcer, which of the following is most likely involved: A. left gastric artery B. gastroduodenal artery C. short gastric artery D. left gastroepiploic artery B 352. Which of the following would inhibit parietal cell acid secretions? A. histamine B. acetylcholine C. gastrin D. prostaglandins D 353. The most common symptom associated with a Meckel’s diverticulum is: A. crampy abdominal pain B. periumbilical pain C. blood per rectum D. watery diarrhea C 354. The typical visual field deficit caused by a pituitary adenoma with suprasellar extension is: A. bitemporal hemianopsia B. homonymous hemianopsia C C. D. superior quadrantanopsia Inferior quadrantanopsia 355. The evaluation of a comatose patient with a head injury begins with: A. evaluation of cardiovascular system B. evaluation of papillary reflexes C. establishment of an airway D. plain film of the skull C 356. The radial nerve is at greatest risk for injury with which fracture: A. fracture of the surgical neck of the humerus B. fracture of the shaft of the humerus C. supracondylar fracture of the humerus D. olecranon fracture B 357. An early sign of compartment syndrome in the hand includes: A. pain with passive stretch at the digits B. absent radial pulse C. motor paralysis D. swelling of the digits A 358. The most common site of involvement for skeletal TB is the: A. tibia B. femur C. pelvis D. spin D 359. A 65 y/o woman with a history of chronic constipation is transferred from a nursing home because of abdominal pain and marked abdominal distension. On examination, her abdomen is found to be distended and tender in the LLQ. The most likely diagnosis is: A. appendicitis B. CA of the colon C. volvulus of the sigmoid colon D. small bowel obstruction C 360. A 68-year old man presents with crampy abdominal pain and distention vomiting. Findings on PE are positive for healed abdominal scars. X-ray reveals multiple gas fluid levels. The WBC count is 12,000. The most likely diagnosis is. A. Small Bowel obstruction due to adhesions B. A Hernia C. Appendicitis D. Gallstones and Ascites A 361. A 56- year old man has suffered from intermittent claudication for 5 years. He has recently developed abdominal pain that is made worse by eating. He has history of a 15-lb weight loss. The most likely diagnosis is: A. Chronic Cholecystitis B. Chronic Intestinal Ischemia C. Peptic Ulcer D. Abdominal Aortic aneurysm B 362. A 28-year old man with a history of emotional disturbance enters the hospital with a history of weight loss and regurgitation of food. Regurgitation is worse when he lies down. The most likely diagnosis is: A. Hiatal Hernia B. Cancer of the esophagus C. Duodenal Ulcer D. Achalasia A 363. A 38-year old man with a history of fever associated with abdominal pain of several weekws duration presents now with a sudden onset of explosive abdominal pain and vomiting. Flat plate x-ray reveals air under the diaphragm. CT scan shows mesenteric lymphadenopathy and splenomegaly. The most likely diagnosis is: A. Tubercolosis enteritis B. Typhoid enteritis C. Primitive peritonitis D. Ulcerative colitis B 364. Splenectomy is often indicated in the management of: A. Hereditary Spherocytosis B. Hereditary neurofibromatosis C. Aplastic anemia D. Hashimoto’s disease A 365. A painless distended gallbladder palpable on PE of jaundice patient is strongly suggestive of: A. Empyema of the gallbladder B. Mirizzi’s Syndrome C. Gallstone impacted on ampulla D. Pancreatic Carcinoma D 366. Which of the ff. suggest unresectability of a left upper lobe lung cancer: A. Hemoptysis B. Malignant pleural effusion C. A cough specimen with positive sputum cytology D. Clubbing and blueness of fingers B 367. Which of the ff. studies is contraindicated in a drowsy patient with papilledema whom one suspects of having closed head trauma: A. Carotid arteriography B. Lumbar puncture C. CT scna D. Echoencephalography B 368. The appropriate antibiotic for a patient with a cellulitis of a leg due to streptococcus is: A. Penicillin B. Erythromycin rd C. 3  generations cephalosporin D. Cloxacillin A 369. Massive bleeding from the lower GI tract (beyond the ligament of Treitz) is most often due to: A. Diverticulosis B. Meckel’s diverticulum C. Diverticulus D. Colonic Carcinoma A 400. A 54-year old male has been complaining of recurrent hypogastric pain for almost 6 months. For almost 3 weeks prior to consult, he had noticed that he is passinf air during micturition. The most common cause of the above condition is: A. Carcinoma of the bladder B. Tubercolosis of the urinary tract C. CA of the sigmoid colon D. Diverticulitis of the colon D 401. A 42-year old woman has been complainging of intermittent gross hematuria. On urinalysis, she has significant pyuria but without bacilluria. The condition is most commonly due to: A. Urethritis B. Bladder stone C. Perinephric abscess D. Tubercolosis of the kidney D 402. A 25-year old male had episodes of severe vomiting due to gastric outlet obstruction. He appears pale and dehydrated. The most likely metabolic anomality the patient would have is: A. Hypochloremic acidosis B. Hyponatremic acidosis C. Hypochloremic alkalosis D. Hypernatremic alkalosis C 403. A 1 week old infant presents wuth moderate respiratory distress and tympany in only one hemithorax. The most likely diagnosis is: A. Spontaneous pneumothorax B. Eventration of the diaphragm C. Bochdalek hernia D. Atelectasis C 404. A little boy aspirated a peanut. He is cyanotic. On expiration, his mediatinum shifts to the right. The peanut most likely is lodged in the: A. Right main stem bronchus B. Left main stem bronchus C. Trachea D. Esophagus B 405. A 40-year old man is hit by a car and sustains an injury to the pelvis. Which of the ff. is most indicative of a urethral injury: A. Hematuria B. High-riding prostrate on rectal examination C. Oliguria D. Scrotal ecchymosis B 406. A 64-year old man has mild upper abdominal pain. In contrast CT-scan, a 5 cm lesion in the left lobe of the liver enhances and then decreases over a 10-minute period from without to within. The most likely lesion is a: A. Hepatic adenoma B. Amoebic abscess C. Fatty infiltration infiltration D. Cavernous hemangioma D 407. A 28-year old male has had a nonseminomatous testicular cancer treated. In followind this patient for possible recurrent tumor, the most useful serum marker would be: A. Carcinoembryonic antigen B. Alpha fetoprotein C. PSA D. Alkaline phosphatase B 408. The most common cause of esophageal rupture or perforation: A. Endoscopic injury B. Blunt chest trauma Boarhaave’s syndrome C. D. Carcinoma A 409. The superior mesenteric artery communicates with the celiac artery via the: A. Pancreatico-duodenal artery B. Splenic artery C. Hepatic aretey D. Dorsal pancreatic artery A 410. The most common offending organism in pyogenic osteomyelitis is: A. Eschericia coli B. Staphylococcus aureus C. P. aeruginosa D. Enterococcus B 411. In patients who developed a documented episode of deep vein thrombosis, the most frequent and significant long term sequel is: A. Claudication B. Recurrent foot infection C. Development of stasis ulcer D. Pulmonary embolism C 412. In an arterial stenosis, the most critical factor is: A. The length of the stenosis B. The cross-sectional area of the stenosis C. The length and area are equally important D. Whether the stenosis is smooth or has an irregular countour B 413. A 60-year old amn with a history of atrial fibrillation is found to have a cyanotic cold right lower extremity. The embolous most probably originated from: A. An atherosclerotic plaque B. Deep vein thrombosis C. Lungs D. Heart D 414. Which of the ff. elements is not a component of venous thrombogenesis: A. Incompetent valves in perforating vein B. Disruption of endothelial intima C. Venostasis D. Hypercoagulability A 415. In a patient suffering from an acute arterial embolic phenomenon. If the ischemia is not relieved, which of these ff. tissues is the first to be irreversibly damaged? A. Muscle B. Nerve C. Fat D. Synovial Membrane A 416. Which of the ff. clinical manifestation may suggest tentorial herniation: A. Contralateral mydriasis B. Decorticate rigidity C. Contralateral hemiparesis D. Ipsilateral hemianopsia C 417. The most common complication of lung abscess is: A. Empyema B. Pneumothorax C. Brochopleural fistula D. Osteomyelitis of the ribs A 418. The most common posterior-superior sulcus chest tumor in a 6-month old child is: A. Neuroblastoma B. Teratoma C. Cystoic hygroma D. Ganglioneuroma A 419. The most common cause of spontaneous pneumothorax is: A. Tubercolosis B. Emphysematous blebs C. Pneumonia D. Bronchial rupture B 420. What substance is released by macrophages in order to activate T helper/inducer (CD4+) lymphocytes? A. Interleukin Interleukin 1 B. Interleukin Interleukin 2 C. Interleukin Interleukin 3 D. Interleukin Interleukin 4 A 421. A 27-yr old hypertensive, diabetic woman is admitted for cadaveric renal transplantation. She is blood type B and has had four transfusions of packed cells over the preceding 6 months. Which of the following factors would preclude transplantation? A. Donor blood type O B. Positive crossmatch C. Two-antigen HLA match with donor D. Blood pressure of 180/100 mmHg B 422. The primary mechanism of action of cyclosporine A is inhibition of: A. Macrophage function B. Antibody production C. Cytotoxic T cell effectiveness effectiveness D. IL-2 production D 423. Which of the following malignancy exhibits a slow growing, late metastasizing characteristic? A. Squamous cell carcinoma B. Basal cell carcinoma C. Melanoma D. Basosquamous carcinoma B 424. Following the Goodsall- Salmon’s rule in fistula in ano, an anteriorly located opening less than 3 cm from the anal opening follows: A. Straight tract B. Curve tract C. No tract D. S-shaped tract A 425. One of the following statements is correct regarding hyponatremia: A. Serum sodium determination would give normal result B. Hypoglycemia may be a cause C. Management could either be water restriction or replacement depending on the cause D. Hyperactive deep tendon reflex is a manifestation C 426. A 60 kg lean male would normally have this amount of body water A. 36 liters B. 24 liters C. 30 liters D. 48 liters A 427. A practical but useful clinical parameter that can be used to assess the adequacy of volume replacement of patients suffering from hypovolemia is: A. Central venous pressure measurement B. Urine output C. Blood pressure D. Pulse Rate B 428. A 42-year old man who has been on prolonged total parenteral nutrition (TPN) administration was noted to have scaly, hyperpigmented lesions over the acral surface of elbows and knees and alopecia. The most likely cause of the condition is: D A. B. C. D. Essential fatty acid deficiency Excess glucose calories Hypomagnesemia Zinc deficiency 429. A 40-year old man is found to have severe metabolic acidosis with a high anion gap. The most likely cause is: A. Diarrhea B. Renal tubular acidosis C. Ureterosigmoidostomy D. Methanol ingestion 430. A 60-year old female, most mastectomy for breast cancer, presents with headache backache and frequent vomiting. She is extremely thirsty and stuporous. The test most likely to identify the cause is: A. Serum calcium determination B. Serum sodium determination C. Serum potassium determination D. Serum glucose determination A 431. The Trendelenburg’s (head down) position is beneficial in the initial management of which type of shock: A. Hypovolemic shock B. Cardiogenic shock C. Septic Shock D. Neurogenic Shock D 432. The most common form of burn is: A. scald burns B. flame burns C. chemical burn D. electrical burn A 433. A 20 year old man has lymphadenopathy behind and inferior to his right ear. Biopsy shows the lesion to be alymphosarcoma. The most likely site of the primary tumor is: A. Floor of the mouth B. Nasopharynx C. Buccal mucosa D. Mediastinum B 434. The most frequent organism in highest density that can be isolated from contamination following colon perforation are: A. Gram positive aerobes B. Gram negative aerobes C. Gram positive anaerobes D. Gram negative anaerobes D 435. The most clinically significant early physiologic abnormality in post traumatic pulmonary insufficiency is: A. Increased physiologic shunt B. Increased dead space C. Cardiogenic pulmonary edema D. Pulmonary fibrosis A 436. Overall bone mass increases up to what age after which there is an overall decrease in bone mass: A. 10-15 years of age B. 15-20 years of age C. 20-25 years of age D. 30-35 years of age D 437. After multiple myeloma, the most common primary malignant bone tumor is: A. Osteosarcoma Ewing’s sarcoma B. C. Chodrosarcoma D. Fibrosarcoma A 355. The best treatment for septic arthritis of the hip is: A. Repeated daily aspiration B. Arthroscopic drainage C. Surgical drainage/arthrotomy with antibiotic therapy D. Antibiotic therapy only C 356. The most common form of inflammatory arthritis: A. Osteoarthritis B. Rheumatoid arthritis C. Septic arthritis B D. Psoriatic arthritis 357. The gold standard of treatment of femoral shaft fractures in adults is: A. Closed reduction and hip spica application B. Open reduction and rearmed intramedullary nailing C. Closed locked antegrade intramedullary nailing D. Plating C 358. Initial treatment of Talpesequinovarus (clubfoot) is: A. Serial casting immediately after birth/time of diagnosis B. Immediate operative treatment C. Dennis Brown splint D. Pavlik harness A 359. A unique feature of immature bone is the capacity to undergo plastic deformation without breaking and to sustain an incomplete fracture also called: A. Nightstick fracture B. Monteggia fracture C. Galleazzi fracture D. Greenstick fracture D 360. Inflammation of the tendons in the first dorsal compartment containing the Abductor pollicis brevis and extensor pollicis brevis tendons is called: A. Carpal Tunnel syndrome B. Trigger finger De Quervain’s Tenosynovitis C. D. Lateral epicondylitis C